UPSC CMS PAPER-1 1997


1. In a contaminated, punctured wound of the leg of a non-immune child of 10, which one of the following measures would give
the best protection against the development of tetanus?
A. Active immunization and antibiotics
B. Active immunization, antibiotics and immobilisation
C. Active immunization, wound excision and primary closure
D. None of the above
Ans. D (Harrison 16th ed., p 842)
The ideal treatment for this child is active immunization with tetanus toxoid, passive immunization with tetanus
immunoglobulin and antibiotics.
Remember the following important points commonly asked in examination:
1. For clean minor wounds tetanus toxoid is recommended as follows:
a. Unknown tetanus immunization history.
b. Have received less than 3 doses of adsorbed tetanus vaccine with the last dose received more than 10 years
ago.
c. Have received 3 doses of fluid (non adsorbed) vaccine.
d. No passive immunization.
2. For contaminated or severe wounds:
a. Passive immunization is given if not vaccinated or partial immunization is done previously.
b. Active immunization is given to all those who have received 3 or more doses of adsorbed toxoid but more than 5 years have elapsed.
c. Rest of the indications are same as that for clean minor wounds.
3. The dose of TIG is 250 units IM in wounds of average severity.
4. The appropriate dose of tetanus antitoxin is 3,000-6,000 units.


2. Gas in soft tissues could be due to infections by:
A. Staphylococcus epidermidis B. Staphylococcus aureus
C. Beta-haemolytic Streptococcus D. Micro-aerophilic Streptococcus
Ans. B and C (Indian Journal of Medical Microbiology, vol. 21, 2003 p 202-204; www. pidj. com)
Apart from Clostridium spp, non clostridial crepitant infections may present as myonecrosis simulating gas gangrene,
with presence of gas in the tissues. Among the aerobic gram negative bacilli, E. coli was predominant followed by
Proteus spp and Pseudomonas spp. Among the aerobic gram positive cocci Staphylococcus aureus, was predominant
followed by Streptococcus pyogenes (beta-haemolytic streptococci). Among the anaerobic gram negative non
sporing bacilli Bacteroides fragilis was present. Recently Klebsiella and Salmonella infections of soft tissues have
been reported to be causing gas in soft tissues.


3. Turban tumour of scalp is:
A. Plexiform neurofibroma B. Basiloid carcinoma
C. Squamous cell carcinoma D. Cylindroma
Ans. D (Bailey and Love 23rd ed., p 175)
Turban tumour of scalp is cylindroma; named due to arrangement of stroma in peculiar transparent cylinders,
which are thought to be of apocrine origin.
Basal cell tumours include basal cell epithelioma, basal cell carcinoma and basiloid tumours.

Remember the following important points about features of turban tumour:
1. Relatively benign.
2. Ulceration is uncommon.
3. Forms extensive turban like swelling extending over scalp.
4. For control of progression cryotherapy is used.

4. Match List-I with List-II and select the correct answer using the codes given below the Lists:
List-I List-II
a. Hodgkin’s disease 1. Adriamycin, mitomycin, 5-fluorouracil
b. Breast carcinoma 2. Cyclophosphamide, methotrexate, 5-fluorouracil
c. Cancer buccal mucosa 3. Cyclophosphamide, oncovin, procarbazine, prednisolone
d. Cancer stomach 4. Cisplatinum, 5-fluorouracil
Codes:
A. a b c d B. a b c d C. a b c d D. a b c d
3 2 1 4 2 3 1 4 3 2 4 1 2 3 4 1
Ans. C (Harrison 15th ed., p 577, 580, 726)
Chemotherapy in CA breast:
􀁺 CMF regimen (modified CMF regimen).
􀁺 Humanized antibody to erb B2 (Herception) combined with paclitaxel.
Harrison states “….Patients treated with adjuvant regimen such as cyclophosphamide, methotrexate and fluorouracil
(CMF) regimen may subsequently respond to same combination in metastatic disease setting….”
CA stomach: Surgery gives only chance of cure.
􀁺 Patients with distal carcinomas: Subtotal gastrectomy.
􀁺 Patients with more proximal carcinoma: Total or near total gastrectomy.
􀁺 Combination chemotherapy: For advanced gastric carcinoma such combination generally include 5-FU and
doxorubicin together with mitomycin C, cisplatin or high doses of methotrexate.
In Hodgkin’s disease, chemotherapeutic regimens used are:
􀁺 Doxorubicin, bleomycin, vinblastine, dacarbazine (ABVD).
􀁺 Mechlorethamine, vincristine, procarbazine and prednisone (MOPP).


5. The intensity of the ‘catabolic phase’ in major burns of the body:
A. Reaches its peak in 48 hours and then declines quickly B. Continues to remain active till the wound heals
C. Falls abruptly after 72 hours D. Falls slowly after 72 hours
Ans. B (Sabiston Textbook of Surgery 16th ed., p 349; Schwartz Principles of Surgery 6th ed., p 251)
After severe burns and resuscitation, hypermetabolism develops, which is characterized by tachycardia, increased
cardiac output, elevated energy expenditure, increased oxygen consumption, massive proteolysis and lipolysis and
severe nitrogen losses.
It is sustained for several months in severe burn injuries leading to massive weight loss and decreased strength.
These alterations in the metabolism are due in part to the release of catabolic hormones, which include catecholamines,
glucocorticoids and glucagon. Cytokines and lipid mediators (which includes prostaglandins, leukotrienes and
platelet activating factor) also contribute to maintain hypercatabolic state. The magnitude of this response is proportional
to the size of thermal injury and increases in a linear fashion with increase in burn size. The increased metabolic
expenditure remains relatively constant over the next few weeks until the burn wound either spontaneously heals or
is closed by skin grafting. The increased metabolic rate persists as long as wound is hyperaemic and immature and
only declines to normal values when the wound is fully mature and blanched.


6. Consider the following statements:
The ratio of ankle to brachial systolic pressure (ABI) reflects:
I. Viability of the diseased limb II. Nonviable limb
III. Blood flow in the ischemic areas of skin IV. Hemodynamic status in relatively large vessels
Of these statements:
A. I, II and III are correct B. II, III and IV are correct
C. I, II and IV are correct D. I, III and IV are correct
UPSC Paper-1 1997 1997.3
Ans. C (Bailey and Love 24th ed., p 923-924)
Ankle to brachial systolic pressure index (ABPI/ABI) is ratio of systolic pressure at ankle to that in the arm. ABPI
is normally 1.
􀁺 Values below 0.9 indicate some degree of arterial obstruction.
􀁺 Value < 0.3 suggest imminent necrosis.
􀁺 Arterial obstruction gives idea about blood flow to areas supplied by artery.
􀁺 Imminent necrosis by < 0.3 ABPI gives information regarding viability or non viability of diseased limb.
ABPI is diagnosed by doppler ultrasound blood flow detection.



7. Two years ago, a 30-year old lady had undergone near total thyroidectomy for papillary carcinoma. During the follow-up
visits, she was found to have multiple, discrete, mobile, left, deep cervical lymph nodes measuring 1 to 1.5 cm in size. The
ideal treatment for this lady would be:
A. Iodine-131 ablation B. External irradiation to the neck
C. Thyroxine therapy D. Modified radical neck dissection
Ans. C (Bailey and Love 23rd ed., p 730; 24th ed., p 801)
Scanning after operation for differentiated carcinoma by using radioiodine is probably only indicated in patients with
unresectable local recurrence or metastatic diseases and in those with a rising serum thyroglobulin level.
If metastasis take up radioiodine they are likely to be suppressed as effectively by treatment with thyroxine as by
radioiodine. According to Bailey and Love “….Cases, in which suppression has failed and radioiodine has given
permanent control appear to be uncommon. Solitary distant metastases may be treated by external radiotherapy.…”
According to 24th ed., Bailey and Love “....It is a standard practice to prescribe thyroxine (0.1-0.2 mg daily) to
suppress endogenous TSH production for all patients after operation for differentiated thyroid carcinoma....



8. The treatment of non-functioning thyroid nodule in a 40-year old male should be:
A. Radiotherapy B. Cortisone
C. Surgical excision D. Large dose of iodine-131
Ans. C (Bailey and Love 23rd ed., p 726; 24th ed., p 784-789)
Surgical excision is the treatment modality among given choices for non-functioning thyroid nodule by wide excision
preferably a lobectomy.
Radioiodine will not be taken up in non-functioning thyroid nodule.


9. A 28-year old lady has lobular carcinoma right breast stage T2 N0 M0. In her case, the ideal treatment would be:
A. Simple mastectomy B. Modified radical mastectomy
C. Lumpectomy with irradiation D. Neoadjuvant therapy followed by lumpectomy
Ans. B (Schwartz Principles of Surgery 6th ed., p 565)
Bilaterality, multifocality and multicentricity are seen in lobular carcinoma of breast.
Non-invasive lobular carcinoma is better known as lobular carcinoma in situ (LCIS). If operation is chosen even for
LCIS anything less than total mastectomy is inappropriate because the disease process is diffuse and often bilateral.


10. An adolescent boy presents himself with bilateral prominence of the breasts and wants the breasts to be removed. Which
one of the following incisions would be ideal?
A. Radial incision B. Incision along the areolar margin
C. Submammary incision D. Elliptical incision
Ans. B (Schwartz Principles of Surgery 6th ed., p 540)
The most effective therapy, especially in young adult is transareolar mastectomy. However in most of the adolescent
boys surgical mastectomy is not required and reassurance is the best treatment.


11. A 50-year old woman complains of intermittent bleeding from the left nipple over the past 3 months. No mass is palpable,
but a bead of blood can be expressed from the nipple.
The ideal procedure in this case would be:
A. Cytological examination of discharge and if no malignant cells, to be kept under careful observation
B. Segmental excision of breast
C. Microdochotomy
D. Simple mastectomy
Ans. A (Bailey and Love 24th ed., p 828)
For treatment of abnormal discharge from nipple first exclude a carcinoma by occult blood test and
cytology. Simple reassurance may then be sufficient, but if the discharge is proving intolerable an operation to
remove the affected ducts or duct can be performed.
Table: Discharges from the nipple (principal causes are italicised).
Discharge from more than one duct:
1. Blood stained: 3. Milk:
– Duct ectasia – Lactation
– Fibrocystic disease – Rare causes:
Carcinoma a. Hypothyroidism
2. Serous: b. Pituitary tumour
– Duct ectasia 4. Grumous: Duct ectasia
– Carcinoma 5. Purulent: Infection
Fibrocystic disease
Discharge from surface:
– Paget’s disease
– Skin diseases (eczema, psoriasis)
– Rare causes (e.g., chancre)
Discharge from a single duct:
1. Blood stained:
Intraductal carcinoma
Intraductal papilloma
2. Serous, any colour:
Fibrocystic disease
Duct ectasia
– Carcinoma


12. A mobile, variegated large lump in the breast of a 20-year old female is most likely to be due to:
A. Medullary carcinoma B. Inflammatory carcinoma
C. Cystosarcoma phylloides D. Lobular carcinoma
Ans. C (Bailey and Love 24th ed., p 834-835)
Mobile, variegated large lump in breast suggests diagnosis of serocystic disease of Brodie/cystosarcoma
phylloides.
Mobile nature of large lump is typically suggestive of benign nature of disease. All other options in question are
features of malignant tumour.
Although cystosarcoma phylloides usually occur in women over the age of 40 but can appear in younger age.
Ulceration may occur due to pressure necrosis. In spite of large size they remain mobile on chest wall.
Histologically resembles fibroadenoma. Despite the name of cystosarcoma phylloides they are rarely cystic and
only very rarely develop features of sarcoma.
Cystosarcoma phylloides may metastasize via the blood stream.
Treatment:
a. Benign tumour:
– Enculeation in very young.
– Wide local excision.
b. Malignant/massive tumour/recurrent tumour: Mastectomy.

13. Splenic vein thrombosis is best treated by:
A. Splenectomy B. Porto-caval shunt
C. Spleno-renal shunt D. Mesenterico-caval shunt
Ans. A (Harrison 15th ed., p 1761)
UPSC Paper-1 1997 1997.5
Splenectomy may be indicated if splenomegaly is the cause rather than the result of portal hypertension as in
splenic vein thrombosis.
According to Bailey and Love 23rd ed., p 942 “….Patients with oesophageal and/or gastric varices secondary to
splenic or portal vein thrombosis can be effectively treated by splenectomy and gastrooesophageal
devascularization.…”


14. Consider the following statements:
A stone in the common bile duct presents with intermittent:
1. Colicky pain 2. Jaundice
3. Enlarged gall bladder 4. Fever with chills
Of these statements:
A. 1, 2 and 4 are correct B. 1, 2 and 3 are correct
C. 3 and 4 are correct D. 1, 2, 3 and 4 are correct
Ans. A (Bailey and Love 24th ed., p 1109)
Colicky pain, jaundice and fever form the triad of clinical features of cholangitis. The gall bladder will be non
palpable as per Courvoisier’s law. See Q 112 paper 1 UPSC 1998 for further explanation.

15. A patient is admitted with severe pain in the abdomen, nausea, vomiting and fever. There is tenderness and abdominal
rigidity. Serum amylase is found to be 1000 IU/litre. The most likely diagnosis is:
A. Perforated peptic ulcer B. Intestinal obstruction
C. Acute pancreatitis D. Acute cholecystitis
Ans. C (Harrison 16th ed., p 1897)
The patient is suffering from acute pancreatitis


16. Preservation of ilioniguinal nerve is an important step during inguinal hernia operation while:
A. Incising the sub-cutaneous tissue B. Incising the external oblique aponeurosis
C. Incising the cremasteric fascia D. Isolating the sac
Ans. B (Bailey and Love 24th ed., p 1277)
After superficial inguinal ring is identified, external oblique aponeurosis is incised in the line of its fibers and the
structures beneath are carefully separated from its deep surface before completing the incision through the superficial
inguinal ring. In this way ilioinguinal nerve is safeguarded.


17. All the following statements are true of incisional hernia except:
A. Common in multiparous female patients
B. More common after paramedian incision than after mid-line incision
C. Common after wound infection
D. Common after tubectomy/hysterectomy/caesarean section
Ans. B (Bailey and Love 24th ed., p 1290)
Midline and vertical incisions have a tendency to burst, which is higher than those, which are transverse. Between
midline and paramedian incision midline has more tendency to burst.
Factors related to the incidence of burst abdomen/abdominal dehiscence and incision/ventral hernia:
– Technique of wound closure.
– Choice of suture material: Catgut has higher incidence of burst abdomen than use of non-absorbable monofilament
polypropylene, polyamide or wire and should be never used.
– Method of closure: Interrupted suturing has a low incidence.


18. Acute mechanical large bowel obstruction should be operated early because of the risk of:
A. Respiratory embarrassment due to abdominal distension
B. Electrolyte imbalance from vomiting
C. Septicaemia from bowel contents
D. Closed-loop obstruction and caecal perforation
Ans. D (Schwartz Principles of Surgery 6th ed., p 1029)

In colonic obstruction, progressive distension, which may be marked in the presence of a competent ileocaecal
valve, is most dangerous aspect of colonic obstruction. If ileocaecal valve is competent then a closed loop obstruction
is present with progressive distension and strong possibility of caecal perforation.
The caecum is the location of perforation because of the law of Laplace, which relates pressure to volume.


19. Maximal reduction in gastric acidity is achieved by:
A. Truncal vagotomy and pyloroplasty B. Truncal vagotomy and antrectomy
C. Partial gastrectomy D. Highly selective vagotomy
Ans. B (Bailey and Love 24th ed., p 1042; Schwartz Principles of Surgery 6th ed., p 702, 707)
Gastrin is largely confined to the gastric antrum so antrectomy and truncal vagotomy is usually associated with
maximum acid reduction.
Patients whose ulcers are resistant to treatment with H2 receptor blockers may have higher rate of ulcer recurrence
after highly selective vagotomy. These may be best treated by vagotomy and antrectomy with accepted difference in
recurrence rates.
– Lowest recurrence rate is seen with vagotomy and antrectomy.
– Most widely used procedure at present is truncal vagotomy plus a drainage procedure.


20. Consider the following drugs:
1. Ranitidine/H2 receptor blockers 2. Somatostatin
3. Pitressin 4. Glucagon
Drugs used in the treatment of bleeding duodenal ulcer would include:
A. 1 and 2 B. 1 and 3
C. 2, 3 and 4 D. 1, 2 and 4
Ans. A (Bailey and Love 24th ed., p 1047; Kumar and Clark 4th ed., p 245)
Though medical treatment has limited efficacy. Patient is commonly started on either H2 antagonists or proton pump
antagonists.
– Tranexaemic acid an inhibitor of fibrinolysis reduces the rebleeding rate.
– Octreotide, a somatostatin analogue (which reduces the splanchnic blood flow as well as acid secretion) can be
given as an infusion if the bleeding is difficult to stop, although a meta-analysis of clinical trials has shown no
clear benefit.


21. A scooter is hit from behind. The rider is thrown off and he lands with his head hitting the kerb. He does not move,
complains of severe pain in the neck and is unable to turn his head. Well-meaning onlookers rush up to him and try to
make him sit up. What would be the best course of action in this situation?
A. He should be propped up and given some water to drink
B. He should not be propped up but turned on his face and rushed to the hospital
C. He should be turned on his back and a support should be placed behind his neck and transported to the nearest hospital
D. He should not be moved at all but carried to the nearest hospital in the same position in which he has been since his fall
Ans. D (Bailey and Love 24th ed., p 554)
Adequate immobilization of patients with unstable cervical injuries is mandatory. The patient should have a hard collar
with sandbags at either side of the head and the head should be taped to the bed until adequate treatment is started.
Immobilization with pads, tapes and a hard collar for transporting all patients is required in whom a clinical spinal
injury is a possibility.

22. In a patient with multisystem trauma, the presence of hypotension along with elevated central venous pressure is
suggestive of:
A. Upper airway obstruction B. Major abdominal bleed
C. Cardio-pulmonary problem D. Spinal cord injury
Ans. C (Bailey and Love 23rd ed., p 830)
The patient has most probably developed cardiac tamponade due to haemopericardium.
If the pressure of the fluid accumulated in pericardial sac exceeds the pressure in atria, compression will occur,
venous return will fall and circulation is compromised. This state of affairs if occurring acutely is called cardiac
tamponade. Emergency treatment is wide bore needle aspiration of pericardial space under local anaesthesia.
UPSC Paper-1 1997 1997.7
Pericardial
effusion
Heart
Pericardium
Fig.: Site of drainage of cardiac tamponade.


23. Lateral cleft lip represents the failure of complete fusion of the:
A. Maxillary and medial nasal processes B. Maxillary and lateral nasal processes
C. Medial and lateral nasal processes D. Two maxillary processes
Ans. A (Bailey and Love 24th ed., p 646; 23rd ed., p 588; 22nd ed., p 423)
Cleft lip results from abnormal development of medial nasal and maxillary process at time they bulge downwards in
front of and below the nasal pit.
Cleft palate results from a failure of fusion of the two palatine processes or in the case of the soft palate, of a
merging process to carry the union backwards from the site of initial fusion.
Treatment:
1. Cleft lip alone:
a. Unilateral (one side): One operation at 5-6 months.
b. Bilateral (both sides): One operation at 4-5 months.
2. Cleft palate alone:
a. Soft palate only: One operation at 6 months.
b. Soft & hard palate: Two operations. Soft palate at 6 months. Hard palate at 15-18 months.
3. Cleft lip and palate:
a. Unilateral: 2 operations. Cleft lip and soft palate at 5-6 months. Hard palate and gum pad with or without lip
revision at 15-18 months.
b. Bilateral: 2 operations. Cleft lip and soft palate at 4-5 months. Hard palate and gum pad with or without lip
revision at 15-18 months

24. The ideal surgical treatment for Pott’s paraplegia is:
A. Laminectomy and decompression B. Anterior decompression and bone grafting
C. Anterolateral decompression D. Costotransversectomy
Ans. B (Maheshwari 3rd ed., p 178-179)
Whenever facilities are available expose the spine from front using transthoracic or transperitoneal approaches and
perform a radical debridement. All the dead and diseased vertebrae are excised and replaced by rib grafts also
known as Hongkong operation.
The advantage is early healing of the disease and no progress of the kyphosis.
Other surgeries used in Pott’s paraplegia:
􀁺 Anterolateral decompression is the most commonly performed operation.
􀁺 Costotransversectomy is indicated in a child with paraplegia and when a tense abscess is visible on X-ray.
􀁺 Laminectomy is indicated in cases of spinal tumour syndrome and those where paraplegia has resulted from
posterior spinal disease.
􀁺 Anterior decompression is preferred in cervical spine tuberculosis.
Remember the following important points commonly asked in examination:
1. Spine is the commonest site of bone and joint tuberculosis.
2. Paradiscal is the commonest type of spinal tuberculosis.

3. Anti-tubercular chemotherapy forms the mainstay of treatment.
4. The spine is put to absolute rest by sling traction for the cervical spine and bed rest for dorsolumbar spine.
5. Pott’s paraplegia is TB spine with neurological involvement. It occurs most commonly in tuberculosis of the
dorsal spine because the spinal canal is narrowest in this part.
6. The onset of paraplegia is gradual in onset in most cases but in some it is sudden.
7. Tubercular paraplegia is usually spastic to start with clonus (ankle or patellar) is the most prominent early sign.
8. Reduction of disc space is the earliest sign in the commoner, paradiscal type of tuberculosis.
9. Reduction of disc space is an important sign because in other diseases of the spine e.g., secondaries in spine,
disc space is well preserved.
10. Back pain is commonest presenting symptom in Pott’s disease.
11. Stiffness is very early symptom in TB spine.
12. TB of spine is always secondary.
13. Commonest cause of early onset paraplegia is abscess.


25. Match List-I with List-II and select the correct answer using the codes given below the Lists:
List-I List-II
a. Volkmann’s contracture 1. Complication of fracture femur
b. Fat embolic syndrome 2. Often associated with supracondylar fracture of humerus
c. Nerve injury 3. Common in fracture tibia
d. Non-union 4. Complication of fracture humerus
Codes:
A. a b c d B. a b c d C. a b c d D. a b c d
1 2 3 4 2 1 3 4 2 1 4 3 1 2 4 3
Ans. C (Maheshwari 3rd ed., p 78, 82, 122, 133)
Complications of fracture tibia
– Delayed and non-union: Especially in fractures of lower 1/3rd.
– Malunion.
– Compartment syndrome.
– Infection.
– Injury to major vessels and nerves: Especially in fractures of tibia in upper 1/3rd of shaft may lead to injury to
popliteal artery.
Complications of fracture shaft of humerus
1. Nerve injury: Radial nerve is the most commonly injured nerve.
2. Delayed and nonunion: Especially seen in transverse fractures of mid shaft.
Causes of non-union are generally inadequate immobilization or distraction at the fracture site because of the effect
of gravity.
Complications of supracondylar fracture of humerus
1. Immediate:
a. Injury to brachial artery: By the sharp edge of the proximal fragment.
b. Injury to nerve: Radial nerve is the most commonly injured nerve.
2. Early: Volkmann’s ischaemia.
3. Late:
a. Malunion: Commonest complication, leading to cubitus varus or gun-stock deformity.
b. Myositis ossificans.
c. Volkmann’s ischaemic contracture (VIC)
Table: Complications of fracture shaft of femur.
Early complications Late complications
– Shock – Delayed union
– Fat embolism – Non union
– Injury to femoral artery – Malunion
– Injury to sciatic nerve – Knee stiffness
– Infection





26. Monteggia fracture dislocation is the fracture of:
A. Ulna and dislocation of superior radioulnar joint
B. Radius and dislocation of inferior radioulnar joint
C. Both bones of the forearm and dislocation of superior radioulnar joint
D. Both bones of the forearm and dislocation of inferior radioulnar joint
Ans. A (Bailey and Love 24th ed., p 378)
Two Important fracture dislocations of forearm are:
– Monteggia fracture: Proximal ulna fracture, associated with dislocation of radial head/superior radioulnar
dislocation.
– Galeazzi fracture: Consists of distal radial shaft fracture with disruption of distal radioulnar joint.


27. When a 11-year old child was siphoning kerosene into a tin, he had a bout of cough. In a few hours he developed severe
respiratory distress and was hospitalised. He was treated with oxygen, antibiotics, steroids and IV fluids. Over the next two
days his condition remained critical. On the third day, his condition deteriorated. Both lung apices were resonant and both
bases were dull to percussion. There was shifting dullness in the chest. Air entry was poor. The temperature varied
between 102°F and 104°F and there was profuse sweating and circulatory collapse. Deterioration on the third day was most
likely due to:
A. Bilateral haemo-pneumothorax B. Bilateral pyo-pneumothorax
C. Bilateral hydrothorax D. Cardiac tamponade
Ans. B (Harrison 16th ed., p 2585)
The boy has developed bilateral pyopneumothorax.
The presence of pus is diagnosed by high fever with sweating and circulatory collapse.
Presence of air and fluid in pleural space is diagnosed by:
a. Lung apices resonant and lung bases dull.
b. Shifting dullness in chest.
c. Poor air entry.
Kerosene poisoning is common in India. Aspiration into lungs causes pneumonia. CXR may reveal infiltrates,
atelectasis, effusions, pneumothorax, pneumatoceles etc.
Treatment is symptomatic and supportive.

28. In majority of patients with oesophageal leaks in thoracic cavity of less than 12 hours duration, the treatment of choice is:
A. Primary closure, drainage and antibiotics B. Early oesophagogastrostomy
C. Exclusion and diversion of continuity D. Total oesophagectomy and gastric pull up
Ans. A (Bailey and Love 23rd ed., p 862; 24th ed., p 998)
According to 24th edition of Bailey and Love treatment is controversial with strong opinion favouring operative and
nonoperative management.
Operative management i.e., thoracotomy and repair of the perforation is best done within a few hours of perforation.
After 12 hours the tissues become swollen and friable, and less suitable for direct suture.
– Oesophageal perforation is most common cause of mediastinitis, which is very dangerous condition.
– First clinical sign of oesophageal perforation is pain.
– Instrumentation is by far the commonest cause of oesophageal perforation.
Management options in perforation of oesophagus:
Factors that favour:
Non operative management Operative repair
– Small septic load – Large septic load
– Minimal cardiovascular upset – Septic shock
– Perforation confined to mediastinum – Pleura breached
– Perforation by flexible endoscope – Boerhaave syndrome
– Perforation of cervical oesophagus – Perforation of abdominal oesophagus.

29. If a patient with a suspected fracture of the pelvis has some bleeding from the urethra and is unable to pass urine:
A. He should be encouraged to pass urine after being given antibiotics and analgesics
B. He should be immediately catheterised in the ward
C. A hot water bottle should be given followed by injection of carbachol
D. He should be prepared for surgery and catheterisation attempted in the OT
Ans. D (Bailey and Love 24th ed., p 1390; Sabiston Textbook of Surgery 16th ed., p 1666-1667)
Patient should not be encouraged to pass urine as it will lead to extravasation of urine. He should not be immediately
catheterized as it will lead to complete rupture of urethra if patient has partial tear.
Hot water bottle and carbechol are of no use.
Patient should be prepared for surgery after radiographic studies and suprapubic cystostomy should be performed
before catheterization is attempted in OT.


30. All of the following are true of Wilms’ tumour except:
A. 80% of the patients are under 4 years of age
B. The tumour rarely crosses the mid-line
C. Bilateral kidney involvement is reported in 5% of the cases
D. It is the commonest cause of kidney swelling in children
Ans. D (Smith General Urology 15th ed., p 391; Harrison 15th ed., p 403)
Nephroblastoma/Wilm’s tumour is most common solid renal tumour in childhood.
􀁺 It is not the most common cause of renal swelling in childhood.
􀁺 According to Nelson multicystic disease of kidney (renal dysplasia) is the commonest abdominal mass in
neonate.
􀁺 Nephroblastoma accounts for 5% of all childhood cancers.
􀁺 No sex predisposition.
􀁺 7% cases are bilateral.
􀁺 Wilms’ tumour is associated with overgrowth syndromes such as:
– Beckwith Wiedman syndrome.
– Isolated hemihypertrophy.
􀁺 Non overgrowth syndromes:
– Aniridia.
– Trisomy 18.
– Genitourinary abnormality e.g., hypospadias, cryptorchidism & renal fusion.
􀁺 Genetic defect associated with Wilms’ tumour is 11p13.
􀁺 Triad of abdominal mass, pyrexia and haematuria are seen.
􀁺 Most common site of metastasis is lung.
􀁺 Hydronephrosis is probably the commonest cause of renal swelling in children.

31. Following injury to exciting eye, sympathetic ophthalmitis begins:
A. In 4 to 5 days B. In 4 to 6 weeks
C. In 8 to 12 weeks D. After 12 weeks
Ans. B (Parsons’ Diseases of the Eye 19th ed., p 417)
Sympathetic ophthalmitis is a condition in which serious inflammation attacks the second eye after injury of the
other. Sympathetic ophthalmitis almost always results from a penetrating wound. Wounds involving ciliary body and
leading to its incarceration in the scar is particularly dangerous. It usually occurs between 4-8 weeks after the injury
to the first eye. The incidence has been reported as early as 9 days and may be delayed for months. The etiology
is unknown but is considered to be an autoimmune, T cell mediated disease. Dalen-Fuchs nodules are present on
the iris and ciliary body. The prodromal symptoms are photophobia and blurring of vision due to weakness of
accommodation. When fully developed signs and symptoms of granulomatous uveitis are present.
Steroids are useful in the treatment if started early.

32. Warthin’s tumour of parotid is:
A. Adenolymphoma B. Pleomorphic adenoma
C. Cylindroma D. Carcinoma

Ans. A (Bailey and Love 23rd ed., p 658)
– Warthin’s tumor (adenolymphoma) occurs only in the parotid gland.
– It is a disease of elderly with a mean age of presentation of 60 years.
– Male:female ratio 4:1 and is related to cigarette smoking.
– Warthin tumour/adenolymphoma is benign and does not undergo malignant change.
– Characteristically Warthin’s tumour gives hot pertechnate scan.
Remember the following important points about parotid tumour:
– For parotid and submandibular tumours CT and MRI are most helpful imaging techniques.
– Only reliable clinical indication of malignancy in parotid tumour is facial nerve palsy in the case of parotid,
induration and/or ulceration of the overlying skin or mucosa and regional lymphatic metastasis.

33. In a case of glottic carcinoma with fully mobile cords, the treatment of choice is:
A. Total laryngectomy B. Radiotherapy
C. Hemilaryngectomy D. Chemotherapy
Ans. B (Dhingra 3rd ed., 373-376)
Treatment of laryngeal cancer:
1. Radiotherapy: Curative radiotherapy for early lesion which neither impairs cord mobility nor invade cartilage/
cervical nodes.
2. Surgery:
a. Conservative surgery:
– Excision of vocal cord after splitting the larynx (cordectomy via laryngofissure).
– Excision of vocal cord and anterior commissure region (partial fronto-lateral laryngectomy).
– Excision of supraglottis i.e., epiglottis, aryepiglotlic folds, false cords and ventricle.
– Conservative surgery is done for selected cases.
b. Total laryngectomy: In cases with:
– Cord-fixation.
– Bilateral cordal lesions.
– Subglottic extension.
– Supraglottic lesions.
– Cartilage invasion with perichondritis.
– Cervical nodes metastasis.
– Failed cases of conservative surgery or radiotherapy.
c. Combined therapy: To decrease the incidence of recurrence.
Remember the following important points about CA larynx:
1. M:F = 10:1.
2. Mostly in age group of 40-70 years.
3. Predisposing factors are: Tobacco, alcohol, previous radiation for benign lesions/laryngeal papilloma,
occupational exposure to asbestos, mustard gas and other chemical or petroleum products.
4. Genetic factors.
5. 3 types of CA larynx:
a. Supraglottic cancer: Nodal metastasis occurs early.
b. Glottic cancer:
– Most common type.
– Most common site is free edge and upper surface of vocal cord in its anterior and middle third.
– Hoarseness is early feature.
– Nodal metastasis is practically never seen in cordal lesions unless the disease spreads beyond the region of
membranous cord.
c. Subglottic cancer:
– Rare lesion.
– Earliest presentation may be stridor or laryngeal obstruction.

34. All of the following are true of submucous resection (SMR) operation for DNS except:
A. Indicated in septal deviation B. Mucoperichondrium is removed
C. Preferably done after 16 years of age D. Done in some cases of epistaxis

Ans. B (Dhingra 3rd ed., p 477)
– Elevation of mucoperichondrium and periosteal flaps is done on both sides.
– Only cartilage and bones are removed.
Indications of sub mucosal resection (SMR) of nasal septum:
– Deviated nasal septum (DNS) causing symptoms of nasal obstruction and recurrent headaches.
– DNS causing obstruction to ventilation of paranasal sinuses and middle ear resulting in recurrent sinusitis and
otitis media.
– Recurrent epistaxis from septal spur.
– As a part of septorhinoplasty for cosmetic correction of external nasal deformities.
– As a preliminary step in hypophysectomy or Vidian neurectomy (Trans-septal approach).
Contraindications to SMR:
– Patients below 17 years of age.
– Acute episode of respiratory infection.
– Bleeding diathesis.
– Untreated diabetes or hypertension.

35. A 25-year old lady sustained a lacerated wound on the back of right thigh by the horn of a bull. The wound was sutured. Two
months later she developed foot drop and an ulcer on the dorsum of the foot. The most likely diagnosis is:
A. Chronic ischaemia to limbs due to popliteal artery injury
B. Partial injury to sciatic nerve
C. Complete division of sciatic nerve
D. Injury to hamstring muscles
Ans. B
The patient has most probably developed partial injury to sciatic nerve. Because of partial injury immediate features
were absent. The common peroneal division of sciatic nerve is probably damaged leading to foot drop and the ulcer
on dorsum of foot is a trophic ulcer.

36. A child presents with ataxia and incoordination. Nystagmus is observed on lateral gaze towards the right side. Areflexia
and hypotonia are also present. The head is tilted towards right side and the child walks with a broad base. The site of the
lesion is:
A. Cerebellum on the right side B. Cerebellum on the left side
C. Brain stem on the left side D. None of the above
Ans. A (Harrison 16th ed., p 140)
The child is suffering from cerebellar lesion of right side.
The various features of cerebellar lesions are:
1. Imbalance: Ataxia, waddling gait, leaning towards side of lesion.
2. Dysarthria: Slurred speech.
3. Nystagmus: Horizontal nystagmus, maximum on looking towards the side of lesion.
4. Incoordination of ipsilateral limbs: Past pointing, intention tremor, impaired alternating movement, decomposition
of movement.
5. Miscellaneous: Hypotonia, decreased tendon reflexes, head tilt and head tremor.

37. Which of the following complications occur due to fracture of floor of anterior cranial fossa?
1. Cerebrospinal fluid rhinorrhoea 2. Anosmia
3. Bilateral black eye
Select the correct answer using the codes given below:
Codes:
A. 1 and 2 B. 1 and 3
C. 2 and 3 D. 1, 2 and 3
Ans. D (Bailey and Love 23rd ed., p 549)
Anterior cranial fossa fracture presents with:
– Subconjunctival haematomas.
– Ansomia.
UPSC Paper-1 1997 1997.13
– Epistaxis.
– CSF rhinorrhoea.
Occasionally:
– Carotico-cavernous fistula.
– Periorbital haematoma or raccoon eye indicates subgaleal haemorrhage not necessarily base of skull fracture.
Middle cranial fossa fracture presents as:
– CSF otorrhoea/rhinorrhoea via Eustachian tubes.
– Haemotympanum.
– Ossicular disruption.
– Battle sign (bruising behind ear).
– VII and VIII cranial nerve palsies.
Important test to screen CSF is to test for beta-transferrin.
Remember the following important points about head injury:
1. Ping-pong fracture/Pond fracture is a smooth depression of the cranial vault usually seen in children.
2. Blow out fracture is caused by fracture of orbital floor with herniation of orbital content and subsequent tethering
of the globe, resulting in pain and diplopia.
3. Extradural haematoma (EDH) occurs usually as a result of squamous temporal bone fractures with laceration
of the middle meningeal artery. Biconvex appearance between dura and bone. Lucid interval is seen. Requires
emergency evacuation.
4. Subdural haematoma (SDH): From torn bridging veins. Concave hyperdense lesion.
5. Chronic subdural haematoma: Most common in infants and adults over 60 years.
– Presents as progressive neurological deficits more than 2 weeks after trauma.
6. Intracerebral bleeding is most commonly due to hypertension.

38. Consider the following criteria:
Ability to:
1. Lift the head for five seconds 2. Open the eyes widely
3. Move the arms 4. Protrude the tongue
Clinical criteria for assessing recovery from neuromuscular blockade would include:
A. 1, 2 and 3 B. 2, 3 and 4
C. 1 and 4 D. 1, 2 and 4
Ans. A (Prys Roberts Brown, 1/91/32)
The clinical diagnosis of adequate postanaesthetic recovery of neuromuscular function is made by the combination
of monitoring techniques and clinical tests.
Best clinical test of adequate neuromuscular function is still head lift (head off the bed for 5 sec).
Other excellent tests are:
– Grip strength.
– Straight leg lift.
– Ability to extend the forearm against resistance.
Experienced anaesthetist will notice other subtle signs of residual weakness such as eyelid lag, weak cough or cry,
whispered speech, inability to show the teeth or clench the jaw or raise the arm at shoulder or to straight leg at hip.
Adequate clinical neuromuscular function should be ensured at end of a procedure by proper intraoperative and
postoperative monitoring, followed by careful clinical observation of patient performance of classical clinical evaluations
described above.

39. At a flow rate of 3 L/min, inspiratory O2 (FIO2) concentration of 24% can be achieved by using a:
A. Nasopharyngeal catheter B. Simple face mask
C. Venturi mask D. Head box
Ans. C (Short Textbook of Anaesthesia 2nd ed., Ajay Yadav p 38-39)
In non-intubated patient oxygen can be delivered by:
1. High flow oxygen delivery system: These work on Venturi principle. The inspiratory gas flows should be 3-
4 times of minute volume. They include Venturi mask and they deliver accurate inspired oxygen (FIO2). Venturi

masks are fixed performance (performance not affected by changes in patient’s tidal volume and respiratory rate)
oxygen delivery devices. These are available in different colours.
Colour Flow rate of O2 (litre per min) FIO2
Blue 02 24%
White 04 28%
Orange 06 31%
Yellow 08 35%
Red 10 40%
Green 15 60%
2. Low flow oxygen delivery system: These are variable performance devices i.e., their performance is
effected by changes in patient’s tidal volume and respiratory rate. Their delivered oxygen accuracy cannot be
predicted. These are very cheap and can be used in wards, preoperative and postoperative rooms and these are
better tolerated by patients as compared to high flow systems. These include:
a. Nasal cannula: The tip of nasal cannula should lie in nasopharynx.
Flow rate (litre/min) FIO2
1 24%
2 28%
3 32%
4 36%
5 40%
6 44%
b. Simple oxygen masks: Very commonly used in wards.
Oxygen flow rate/min Delivered oxygen (FIO2 )
5-6 litres 40%
6-7 litres 50%
7-8 litres 60%
c. Oxygen tents (hoods): Used for children. A lot of wastage of oxygen do occur and accurate concentration cannot
be predicted.
d. Nonrebreathing masks: Can deliver up to 80% oxygen.
e. Rebreathing mask: When fitted tightly they can provide approximately 100% oxygen.
After extensive searching of literature we have kept the answer as choice C. Dear readers if you think otherwise
please contact the publisher with reference.

40. Which one of the following statements is true of undescended testis?
A. Usually descends spontaneously at puberty B. Orchiopexy to be done if no descent by puberty
C. Has a higher incidence of malignancy D. Maintains normal sperm production
Ans. C (Bailey and Love 24th ed., p 1404)
􀁺 About 50% of undescended/incompletely descended testes reaches into the scrotum during the first month of
life but full descent after that is uncommon.
􀁺 Incompletely descended testes are often normal until the age of 6 years but by puberty testis are flabby and
poorly developed as compared with its intrascrotal counterpart.
􀁺 By the age of 16 years irreversible changes have occurred which halts spermatogenesis and limits the production
of androgens to around half of normal output.
􀁺 Orchidopexy is not done before the age of 2 years but testis should be brought down into the scrotum before the
boy starts school.
Hazards of incompletely descended testis are:
– Sterility in bilateral cases – Epididymo-orchitis
– Pain due to trauma – Atrophy
– Associated indirect inguinal hernia – Increased liability to malignant change
– Torsion
UPSC Paper-1 1997 1997.15
Sites of ectopic testis are:
– At superficial inguinal ring.
– In the perineum.
– At the root of the penis.
– In the femoral triangle.
Maternal chorionic gonadotrophin stimulates growth of the testis and may stimulate its migration.

41. A 12-month old male child suddenly draws up his legs and screams with pain. This is repeated periodically throughout the
night interspersed with periods of quiet sleep. When seen after 12 hours, the child looks pale, has just vomited and passed
thin blood-stained stool; there is a mass around umbilicus. What is the most likely diagnosis?
A. Appendicitis B. Intussusception
C. Gastroenteritis D. Round worm obstruction
Ans. B (Bailey and Love 24th ed., p 1195)
The boy is suffering from intussusception. Intussusception classically in an otherwise fit and well male child develops
suddenly as onset of screaming associated with drawing up of legs.
– Attack lasts for a few minutes, recurs every 15 minutes and becomes progressively severe.
– During attack child has facial pallor while in between episodes he is listless and drawn.
– Vomiting may/may not occur.
– Stool normal initially while later ‘red currant jelly stool’ (blood and mucus) are evacuated.
– Classically abdomen is not distended; a lump may be felt in only 50-60% of cases.
Sign of dance is associated with feeling of emptiness in right iliac fossa.
Sausage shaped lump with concavity towards the umbilicus is typical physical sign.
– Most common type is ileocolic.
On plain X-ray evidence of small or large bowel obstruction with an absent caecal gas shadow in ileoileal or ileocolic
cases is seen.
Barium enema is used to diagnose an ileocolic or colocolic form (claw sign).
Equivocal cases of ileoileal intussusception may be evaluated by CT scan, which should reveal the presence of
small bowel mass.
– Barium enema may be used therapeutically.
– Hydrostatic reduction is contraindicated in presence of obstruction, peritonism, or a prolonged history (greater
than 48 hours).
– An important measure during surgical reduction is by squeezing the most distal part of the mass in a cephalad
direction.
– Pulling is never done.
– Difficult cases are reduced by Cope’s method.

42. A four-week old baby presents with jaundice and clay-coloured stools. Which of the following should be included in the
differential diagnosis?
1. Rh factor incompatibility 2. Neonatal hepatitis
3. Choledochal cyst 4. Extra-hepatic biliary atresia
5. Intra-hepatic biliary atresia 6. Hepatoblastoma
Select the correct answer using the codes given below:
Codes:
A. 1, 2 and 3 B. 1, 2, 3, 4, 5 and 6
C. 2, 3, 4 and 5 D. 4, 5 and 6
Ans. C (OP Ghai 6th ed., p 174)
– Rh isoimmunization (erythroblastosis fetalis) is a major cause of severe hyperbilirubinemia. Bilirubin produced in
utero is cleared by the maternal circulation but jaundice can be noticed as early as 30 minutes after birth.
Presentation as late as 4 weeks is not seen.
– Hepatoblastoma is not seen in 4-week-old infant.
– In choledochal cyst, extrahepatic and intrahepatic biliary atresia prime sign is persistent jaundice. Many times
icterus appears to be a continuation of physiological jaundice. In other babies jaundice may not be observed till
two or three weeks of life.
– Neonatal hepatitis presents as conjugated hyperbilirubinemia at 4-6 weeks of age.





43. Following vasectomy for family planning, a patient should be advised to use some other method of contraception, till:
A. Removal of all sutures B. Pain completely subsides
C. Two weeks D. Eight weeks
Ans. D (Harrison 15th ed., p 305; Bailey and Love 24th ed., p 1195)
As after vasectomy the development of azoospermia may be delayed for 2 to 6 months, other forms of contraception
must be used until 2 sperm free ejaculate provide proof of sterility. Among the choices given in question 8 week is
the best answer.
According to Bailey and Love 23rd edition, after vasectomy couple should continue with their normal contraceptive
precautions until the success of the operation has been confirmed by semen analysis performed at 12-16 weeks
after surgery.
– Vasectomy is painless and easily performed under local anesthesia.
– Success rate of recanalization after vasectomy is 60-80% and is possible if the operation is performed within 3-
4 years of vasectomy.
According to Essentials of Gynaecology 1st ed., Ashok Kumar p 30 back up contraceptive method for prevention of
pregnancy is required for a minimum period of 12 weeks postvasectomy or minimum of 20 ejaculations.

44. Central venous pressure monitoring is helpful in:
A. Regulating the speed and amount of fluid infusion B. Regulating the dose of noradrenaline
C. Deciding the need for plasma infusion D. Deciding the requirement for blood transfusion
Ans. A (Harrison 16th ed., p 1306)
The right internal jugular vein is used for estimation of CVP. The sternal angle is used as reference point because
the centre of right atrium lies 5 cm below the sternal angle. The vertical distance between the top of the oscillating
venous column and the level of the sternal angle is determined which is usually less than 3 cm. Hence 3 cm + 5 cm
= 8 cm of blood.
Now if patient is given excess fluid then the CVP will be more than 8 cm of blood. Hence CVP helps in monitoring
speed and amount of fluid infusion.

45. Which of the following are athermal effects of diathermy?
I. Cutting and coagulation II. Eddy currents
III. Piezoelectric effects IV. Diamagnetism of some molecules
Select the correct answer using the codes given below:
Codes:
A. I, II and III B. I, III and IV
C. I, II and IV D. II, III and IV
Ans. D (Bailey and Love 23rd ed., p 1323)
In surgical diathermy high frequency AC current is passed and it liberates heat. Temperature rises around 1000°C
and above. Current frequencies in the range of 400 kHz-10 mHz are used. There are two types of diathermy –
monopolar and bipolar.
Directions: The following seven items consist of two statements, one labelled the ‘Assertion A’ and the other labelled the
‘Reason R’. You are to examine these two statements carefully and decide if the Assertion A and the Reason R are
individually true and if so, whether the Reason is a correct explanation of the Assertion. Select your answers to these items
using the codes given below and mark your answer sheet accordingly.
Codes:
A. Both A and R are true and R is the correct explanation of A
B. Both A and R are true but R is not a correct explanation of A
C. A is true but R is false
D. A is false but R is true

46. Assertion A : Terminal pulp space infection of index finger must be treated urgently.
Reason R : Delay in the treatment of such an infection can lead to acute tenosynovitis.
Ans. B (Apley’s System of Orthopaedics and Fractures 8th ed., p 351; Maheshwari 3rd ed., p 90)

Pulp space infection (whitlow or felon) leads to intense pain because of extremely tight arrangement of fat and
fibrous septa in pulp space. At its proximal end, this space is closed by a septum of deep fascia connecting the
distal flexor crease of finger to the periosteum just distal to the insertion of the profundus flexor tendon. Terminal part
of the digital arteries and nerves traverse this space.
If the condition is recognized very early, antibiotic treatment and elevation of hand suffices. But once an abscess
has formed, the pus must be released through a small incision over the site of maximum tenderness. If treatment is
delayed, infection may spread to the bone, joint or flexor tendon sheath. This artery supplying the shaft of distal
phalanx passes through the pulp space. In infections, the diaphyseal artery is prone to thrombosis resulting in
necrosis of shaft of distal phalanx and sequestrum formation.
It can also lead to pyogenic arthritis of the distal interphalangeal joint and very rarely spread of infection to flexor
tendon sheath (suppurative tenosynovitis) occurs.


47. Assertion A : Injection sclerotherapy is a good treatment of bleeding oesophageal varices due to portal hypertension.
Reason R : Injection sclerotherapy reduces the portal venous pressure.
Ans. C (Harrison 16th ed., p 1864)
Endoscopic sclerotherapy is a very effective method of treatment of acute bleeding from oesophageal
varices. By this method sclerosing agents are directly injected into the varices under direct vision during endoscopy.
The veins are thus sclerosed and bleeding is stopped. This method does not reduce portal hypertension.
Portal hypertension can be reduced by beta-blockers (propranolol) or by creating portal systemic shunts. Nonselective
shunts decompress the entire portal system while selective shunts decompress only the varices.

48. Assertion A : Appendicectomy should not be done in the presence of appendicular lump.
Reason R : Appendicectomy is difficult, dangerous and may lead to the formation of faecal fistula.
Ans. A (Bailey and Love 23rd ed., p 1090)
Management of appendicular lump
In appendicular lump inflammatory process is localized and surgery is difficult and may be dangerous as it may be
impossible to find the appendix and occasionally, a faecal fistula may form.
Standard treatment of appendicular lump when condition of patient is satisfactory is Ochsner-Sherren regimen.
Clinical improvement is visible within 24-48 hours at which time the nasogastric tube can be removed and oral fluids
introduced.
– Failure of mass to resolve should raise suspicion of a carcinoma or Crohn’s disease.
– It is advisable to remove the appendix usually after an interval of 6-8 weeks.
Remember the following important points about appendicitis:
– Most common position of appendix is retrocaecal (74%).
– Appendicular artery is a branch of ileocolic artery.
– Accessory appendicular artery is a branch of posterior caecal artery.
– Appendicitis (acute) is due to mixed growth of aerobic and anaerobic organism.
– In majority of cases some form of obstruction by either a faecolith or stricture is found.
– Intestinal parasite particularly Oxyuris vermicularis (pinworm) can proliferate and occlude the lumen in appendicitis.
Clinical features in appendicitis Clinical signs in appendicitis
Periumbilical colic Pyrexia
Pain shifts to right iliac fossa Localized tenderness in right iliac fossa
Anorexia Muscle guarding
Nausea Rebound tenderness
Signs to elicit in appendicitis:
– Pointing sign
– Rovsing’s sign
– Psoas sign
– Obturator sign/Zachary Cope test

Appendicectomy is treatment of choice for acute appendicitis.
Postoperative complications of appendicectomy are:
– Wound infection – Respiratory complications (rare) – Faecal fistula
– Intra-abdominal abscess – Venous thrombosis and embolism – Adhesive intestinal obstruction
– Ileus – Portal pyaemia (pyelophlebitis) – Right inguinal hernia due to injury
to iliohypogastric nerve.

49. Assertion A : Projection of vestibulospinal pathway to cerebral cortex via thalamus assists in the maintenance of
postural stability.
Reason R : Somatosensory system (from skin, joint and muscles) and visual system (from retina) compensates any
deficiency of vestibulospinal pathway.
Ans. B
Vestibulospinal pathway helps in maintenance of postural stability and somatosensory pathway also helps in
maintenance of stability and it also compensates for any deficiency of vestibulospinal pathway but reason is not the
correct explanation of assertion.

50. Assertion A : Hyperuricaemia is a feature of renal failure, but the patients do not develop acute gout.
Reason R : Uraemic patients exhibit diminished inflammatory response to urate crystals.
Ans. A (Robbins 6th ed., p 979; Harrison 16th ed., p 1651, 1655; www.doi.wiley.com)
Uric acid retention is common feature of chronic renal disease but rarely leads to symptomatic gout. Treatment is
usually not necessary. Here both the statement and the reason are right. There is reduced secretion of proinflammatory
cytokines by monosodium urate crystal stimulated monocytes in CRF.
Besides this another possible reason is that probably these patients have not incurred sustained hyperuricaemia
long enough to develop gout.

51. Assertion A : In meiosis, chiasmata are bridges between correspondingly paired homologous chromosomes.
Reason R : Genetic recombination results in deletion of some portion of paired homologous chromosomes.
Ans. A (Essentials of Clinical Anatomy 1st ed., Ramesh Babu p 43)
At the onset of meiosis, genetic material is exchanged between the paired chromosomes, which form crossover
structures known as chiasmata. This process of DNA recombination is critical in generating the variation that
enables species to adapt to their environment.

52. Assertion A : Acute infection with Toxoplasma gondii is determined by the simultaneous presence of IgG and IgM
antibodies in the serum.
Reason R : IgG antibodies appear only during the acute phase of infection.
Ans. C (Harrison 16th ed., p 1247)
T. gondii rapidly induces detectable levels of both IgM and IgG antibodies in serum. IgM antibody indicates
acute infection. The polyclonal IgG antibodies are parasiticidal in vitro in presence of serum complement and are the
basis of Sabin-Feldman dye test.
Diagnosis of acute infection can be established by detection of simultaneous presence of IgG and IgM titre
that can be detected as early as 2 to 3 weeks after infection. However it peaks around 6-8 weeks and slowly
declines to a baseline level that persists for rest of the life. IgM rises in acute infection and falls early.
A rise in IgG titre without an increase in IgM titre suggests that infection is present but it is not acute. It
means that IgG titre rises during acute phase of Toxoplasma infection but it is raised not only in acute stage but in
later stages of infection also. Isolated rise of IgG titre in later stages of infection is present. It is necessary to
measure the serum IgM titre in concert with IgG titre to better establish the time of infection.

53. Balloon valvotomy is successful in all of the following cases except:
A. Congenital pulmonary stenosis B. Calcified mitral stenosis
C. Mitral stenosis in pregnancy D. Congenital aortic stenosis
Ans. B (Harrison 16th ed., p 1393)
Balloon valvotomy is not indicated in calcified mitral stenosis. It is done in symptomatic patients of MS where the
effective orifice is less than approximately 1 cm2/m2 body surface area.

Mitral valvotomy is done by two methods – balloon valvotomy and surgical (open) valvotomy. In balloon valvotomy a
catheter is directed into the left atrium after puncturing the interatrial septum and then Inoue balloon is directed
across the valve and inflated in the orifice of mitral valve. It is done in uncomplicated MS.
In calcified mitral stenosis open valvotomy is done under cardiopulmonary bypass and deposits of calcium are
removed.
In congenital aortic and pulmonary stenosis balloon valvotomy is done to relieve the symptoms because in these
patients ultimately valve replacement is required. Valve replacement is done in adults usually and not in children.
During pregnancy mitral valvotomy can be done if patients with MS do not respond to conservative measures.
Criteria for mitral valvotomy:
a. Significant symptoms.
b. No mitral regurgitation.
c. Pure MS.
d. LA free of thrombus.
e. Mobile and noncalcified valve.

54. Temporal profile of detection of the given serum enzymes in acute myocardial infarction is:
A. CPK, SGOT, LDH B. SGOT, CPK, LDH
C. CPK, LDH, SGOT D. SGOT, LDH, CPK
Ans. A (Kumar and Clark 4th ed., p 695; Practical Clinical Biochemistry 1st ed., DS Sheriff p 110)
After myocardial infarction several enzymes are raised in circulation, which are released by the necrotic cardiac
cells. The temporal pattern of protein release is of diagnostic importance and can be measured in serum. These
enzymes are creatinine phosphokinase, SGOT, LDH etc.
CPK starts to rise at 4-6 hours, peaks at 24-36 hours and falls to normal in 48-72 hours.
SGOT starts to rise at about 6-12 hours after infarction, reaches a peak on 1st or 2nd day and then declines to
normal in 4-7 days.
LDH starts to rise after 12 hours, reaches a peak after 2 or 3 days and may remain elevated for a week.
CPK is also present in skeletal muscle and hence measurement of CPK-MB (selectively present in cardiac cells) is
more specific for diagnosis of acute MI. CPK-MB begins to rise in 3 to 4 hours, reaches maximum at 12 to 24 hours
and returns to normal in 1 to 3 days.
The importance of measurement of LDH lies in the fact that it helps in diagnosis of MI when the patient comes late
to the doctor. However now it has been replaced by measurement of cardiac troponin-T. It is not normally
detectable in blood but after MI the level is increased more than 20 times and remains elevated for 10-14 days.

55. Consider the following features:
1. Kussmaul’s sign 2. Regurgitant murmurs
3. Ascites out of proportion of oedema 4. Atrial fibrillation
The features of constrictive pericarditis would include:
A. 1 and 4 B. 2, 3 and 4
C. 1, 3 and 4 D. 1, 2 and 3
Ans. C (Harrison 16th ed., p 1419)
Constrictive pericarditis occurs after healing of acute fibrinous or serofibrinous pericarditis. There is
obliteration of pericardial cavity with formation of granulation tissue.
The commonest cause in India is tuberculosis. The other causes are trauma, mediastinal irradiation, collagen
vascular disease, neoplasia etc.
The clinical features are fatigue, weakness, abdominal distension, oedema, dyspnoea, etc. JVP is raised and
Kussmaul’s sign is present. Pulsus paradoxus is seen in 1/3rd cases.
Pericardial knock (0.09 to 0.12 s after aortic valve closure) is the auscultatory hallmark. Murmurs are absent.
Ascites out of proportion to oedema is a very important feature of constrictive pericarditis. Often it resembles cirrhosis.
ECG shows low voltage complex and AF is present in about 1/3rd of patients.
CXR shows pericardial calcification.
Echo shows pericardial thickening. Ventricular pressure pulse shows square root sign during diastole.
Treatment is surgical removal of thickened pericardium.


56. Match List-I (Drug) with List-II (Toxicity) and select the correct answer using the codes given below the Lists:
List-I List-II
a. Amiodarone 1. Thrombocytopaenia
b. Quinidine 2. Pulmonary fibrosis
c. Disopyramide 3. Dryness of mouth
d. Propranolol 4. Bronchospasm
Codes:
A. a b c d B. a b c d C. a b c d D. a b c d
2 1 3 4 2 1 4 3 1 2 3 4 1 2 4 3
Ans. A (Harrison 16th ed., p 1356, 1558)
Drug Side effect
Disopyramide Myocardial depression, hypertension, dry mouth, urinary retention, visual disturbance, constipation.
Propranolol Myocardial depression, bradycardia, bronchospasm, fatigue, depression, nightmares, cold peripheries.
Amiodarone Photosensitivity, skin discolouration, corneal deposits, thyroid dysfunction (both hyper and hypo), alveolitis,
pulmonary fibrosis, nausea, vomiting, hepatotoxicity, peripheral neuropathy, Torsades de pointes.
Quinidine GI upsets, myocardial depression, Torsades de pointes, haemolytic anemia, thrombocytopenia,
cinchonism.


57. A 50-year old male has precordial pain for four hours. On examination, his BP is 110/80 mmHg, pulse is 120 beats/min
and respiratory rate is 26/min. His ECG shows marked S-T segment elevation in leads V3 -V6 and left ventricular ectopics.
The initial therapeutic modalities in his case would include:
A. Lignocaine and streptokinase B. Streptokinase and morphine
C. Morphine and dobutamine D. Lignocaine, streptokinase and morphine
Ans. B (Harrison 16th ed., p 1452-1453)
The patient is suffering from acute MI. The points in favour of diagnosis are:
1. S-T segment elevation in leads V3-V6.
2. Pain chest.
3. 50 years of age.
4. Male.
The drugs required for treatment are:
1. Morphine: 5-10 mg SC or IV is given to relieve chest pain.
2. Streptokinase: 1.5 million units is given IV over 1 hour. It is one of the most commonly used thrombolytic agent.
It is most effective when given within 6 hours of onset of chest pain.
3. Lignocaine: 100 mg is given IV bolus to control ventricular tachycardia/ventricular ectopic. In this question the
frequency of ventricular ectopic is not mentioned. In the initial management lignocaine may not be required
because patient has haemodynamic stability. Besides that thrombolysis itself may abolish VPC. If VPC persists
and cause hemodynamic instability then lignocaine is also indicated in management.
A patient of MI is also treated with:
1. O2 inhalation.
2. Furosemide: Intravenous furosemide (20-40 mg) is given to control LVF.
3. ACE inhibitor: It is given to reduce preload, after load and to prevent ventricular remodeling.
4. Beta blocker: Reduces myocardial oxygen demand.
5. Clopidogrel: Antithrombotic agent.
6. Acetylsalicylic acid: Prevents thrombus formation.


58. Match List-I with List-II and select the correct answer using the codes given below the Lists:
List-I (Radiological signs) List-II (Clinical conditions)
a. Hilar dance 1. Tetralogy of Fallot
b. Rib notching 2. Mitral stenosis
c. Kerley B lines 3. Ostium secundum atrial septal defect
d. Dextroposition of aorta 4. Coarctation of aorta

Codes:
A. a b c d B. a b c d C. a b c d D. a b c d
2 4 3 1 3 1 2 4 3 4 2 1 4 1 3 2
Ans. C
Hilar dance is seen in fluoroscopy in ostium secundum ASD.
Kerley B lines are seen in MS. These are dense, fine, opaque, horizontal lines seen in CXR. These lines are most
prominent in lower and midlung fields and result from distension of interlobular septa and lymphatics with oedema
when the mean resting pressure in LA exceeds 20 mmHg.
Notching of inferior border of ribs (3rd to 8th ribs) is seen in coarctation of aorta. This is due to erosion by dilated
intercostal arteries. It becomes apparent between 4th and 12th years of life. Dilatation of internal mammary artery
may cause scallop-edged retrosternal notching.
Arch of aorta
50% have bicuspid valve
Coarctation of aorta
Internal mammary artery
Intercostal artery
Fig.: Coarctation of aorta.
RV
VSD
RA
Aorta
LA
LV
Infundibular
obstruction
Fig.: Fallot's tetralogy.
The four components of TOF are pulmonary stenosis, VSD, right ventricular hypertrophy and overriding of VSD by aorta.


59. Which one of the following is of most serious prognostic significance in a patient of essential hypertension?
A. Diastolic blood pressure greater than 130 mmHg B. Transient ischemic attacks
C. Left ventricular hypertrophy D. Papilloedema and progressive renal failure
Ans. D (Harrison 16th ed., p 1469)
Papilloedema and progressive renal failure are of most serious prognostic significance in a patient of essential
hypertension.
Accelerated or malignant hypertension diagnosis is based on evidence of high blood pressure and rapidly progressive end organ damage such as retinopathy (grade 3 and 4), renal failure and/or hypertensive encephalopathy.
Prior to the availability of antihypertensive therapy, the life expectancy was less than 2 years after diagnosis of
malignant hypertension.
Malignant hypertension is a medical emergency that requires immediate treatment.


60. In bronchial asthma, glucocorticoids:
A. Act as potent bronchodilators B. Reduce airway inflammation
C. Inhibit degranulation of mast cells D. Block the action of humoral mediators
Ans. B (Harrison 16th ed., p 1513)
Glucocorticoids help in reduction of airway inflammation.

61. Recognised features of asbestosis include all of the following except:
A. Finger clubbing B. Pleural calcification
C. A restrictive pattern of ventilatory defect D. Premalignant collagenous plaques on the pleura
Ans. D (Harrison 16th ed., p 1522)
Asbestosis is probably the commonest inorganic dust related chronic pulmonary disease. Asbestos is general term
that includes several mineral silicates like chrysolite, amosite, crocidolite, anthrophyllite etc.
The major effect of exposure is pulmonary fibrosis. Progressive fibrosis leads to restrictive ventilatory defect.
Benign pleural plaques and pleural effusion are often observed. The plaques are identified in CXR. They are often
calcified and most commonly seen on diaphragm and anterolateral pleura. These plaques do not give rise to cancer.
Squamous cell CA or adenocarcinoma are the commonest cancer associated with asbestos exposure.
Mesothelioma of pleura (rarely peritoneum) is associated with exposure to asbestos.



62. Alveolar air in severe pulmonary hypoventilation contains:
A. High PCO2 and high PO2 B. High PCO2 and low PO2
C. Low PCO2 and high PO2 D. Low PCO2 and low PO2
Ans. B (Harrison 16th ed., p 1571)
By definition alveolar hypoventilation exists when arterial PCO2 increases above the normal range of 37-43
mmHg. However the hallmark of alveolar hypoventilation is increase in alveolar PCO2 (PACO2). The increase
in PACO2 causes an obligatory decrease in PAO2 and hence PO2. Hypercapnia with hypoxaemia results in
polycythaemia.
There are several causes of alveolar hypoventilation like:
a. Impaired respiratory drive: Carotid body dysfunction, metabolic alkalosis, bulbar polio, brainstem infarction,
drugs etc.
b. Defective respiratory neuromuscular system: Polio, high cervical cord injury, MND, peripheral neuropathy, myopathy
etc.
c. Impaired ventilatory apparatus: Kyphoscoliosis, obesity, cystic fibrosis, obstructive sleep apnoea, ankylosing
spondylitis etc.


63. Heart-lung transplantation is indicated in:
A. Primary pulmonary hypertension B. Dilated cardiomyopathy
C. Severe emphysema with respiratory failure D. End-stage interstitial lung disease
Ans. A (Harrison 15th ed., p 1532)
Majority of the heart lung transplant patients are suffering from either primary or secondary pulmonary hypertension
(54%) or cystic fibrosis (16%). Majority of heart lung recipients have concomitant left ventricular disease and end
stage lung disease or irreparable congenital heart disease with Eisenmenger syndrome.


64. The physiological dead space is increased when:
A. Ventilation perfusion ratio is greater than normal B. Ventilation perfusion ratio is less than normal
C. Hypoxia is present D. Cyanosis is present
Ans. A (Short Textbook of Anaesthesia Ajay Yadav 2nd ed., p 4; Kumar and Clark 4th ed., p 752)
For efficient gas exchange ventilation should be distributed uniformly to different parts of the lungs and should be
matched by uniform distribution of blood flow. To assess this three-compartment model of gas exchange has to be understood.

a. Ideal compartment Normal ventilation:blood flow ratio.
b. Venous admixture effect Ventilation:blood flow ratio (physiological right to left shunt) is decreased.
c. Physiological dead space Ventilation:blood flow ratio is increased.
Physiological dead space means that space where ventilation (air) is present but perfusion (blood) is absent and
hence exchange of gas will not be possible. Hence physiologic dead space will increase with increase in ventilation:
perfusion ratio. It is 60 to 80 ml in standing position and zero in lying position (in lying position perfusion is equal in
all parts of lung). It is increased in lung pathologies affecting diffusion at alveolar capillary membrane like interstitial
lung disease, pulmonary embolism, pulmonary edema and ARDS; general anaesthesia; IP PV (intermittent positive
pressure ventilation); PEEP (positive end expiratory pressure) and hypotension.
A certain part of tidal volume (approximately 30%) does not reach the alveoli but remain in the conducting airways.
This is called anatomic dead space component. It is increased in old age, neck extension, jaw protrusion,
bronchodilators, increasing lung volume, anaesthesia mask and circuits, IPPV and PEEP. It is decreased in
intubation, tracheostomy, hyperventilation, neck flexion and bronchoconstrictors.



65. Which of the following are the causes of chronic respiratory failure?
1. Diffuse pulmonary fibrosis 2. Atlantoaxial cervical anomalies
3. Muscular dystrophy 4. Thoracoplasty
Select the correct answer using the codes given below:
Codes:
A. 1, 2 and 3 B. 2, 3 and 4
C. 1, 3 and 4 D. 1 and 4
Ans. C
For definition of Type II respiratory failure see Q 39 paper 1 UPSC 1998.
Atlantoaxial cervical anomalies do not produce respiratory failure. Injury of high cervical cord will produce acute type II respiratory failure.


66. The Schilling test performed in a patient demonstrated reduced vitamin B12 absorption. The test was repeated after
addition of enough of intrinsic factor and then after tetracycline therapy. But the Schilling test again showed reduced
vitamin B12 absorption. Which one of the following is the most likely cause of reduced B12 absorption?
A. Addisonian pernicious anaemia B. Gastrectomy
C. Tuberculosis of terminal ileum D. Bacterial colonisation of small intestine
Ans. C (Harrison 16th ed., p 1769)
Schilling test is performed to determine the cause of vitamin B12 deficiency. It is performed by giving 58Co
labelled cobalamin orally and then 24 hour urine is collected. One hour after consumption of oral cobalamin 1 mg
cobalamin (non radioactive) is injected intramuscularly. Schilling test is abnormal if urinary radiolabelled
cobalamin is <10% of injected dose.
Schilling test is abnormal in all the conditions mentioned in the question but it is corrected with intrinsic factor in
Addisonian pernicious anaemia and gastrectomy as well as in bacterial colonization after tetracycline therapy.
Tuberculosis of terminal ileum will not be corrected by tetracycline. Hence the patient is most likely suffering
from tuberculosis of terminal ileum because cobalamin is absorbed from terminal ileum.


67. Acute liver injury associated with hepatitis B virus is due to:
A. Direct cytopathic effect of the virus B. Sensitized cytolytic T-cells
C. Immune-complex mediated tissue damage D. Vasculitis
Ans. B (Harrison 16th ed., p 1827)
Existence of asymptomatic hepatitis B carriers with normal liver histology and function suggests that the virus is not
directly cytopathic.
Cellular immune response plays a major role in pathogenesis of hepatitis B related liver injury. Latest
observations suggest that nucleocapsid proteins (HBcAg and possibly HBeAg) present on cell membrane in minute
quantities are the viral target antigens that with host antigens invite cytolytic T cells to destroy HBV infected liver
cells.







68. The biochemical findings on sweat analysis in a patient with cystic fibrosis would include:
A. High sodium and high chloride B. Normal sodium and low chloride
C. Low sodium and low chloride D. Low sodium and high chloride
Ans. A (Harrison 16th ed., p 1545)
Cystic fibrosis is an autosomal recessive disease. It is due to mutation in a gene located on chromosome 7. This
results in absence of phenylalanine at amino acid position 508 of gene product known as CF transmembrane
regulator.
Both Na+ & Clis high in sweat because in these patients Na+ is not absorbed from sweat as it moves through
sweat duct due to inability to absorb Clacross the ductal epithelial cells.
In children sweat Cl>60 mmol/L and in adults >70 mmol/L on two occasions are diagnostic of cystic fibrosis.
The children suffer from upper and lower respiratory tract infections frequently. Initially H. influenzae and S. aureus
are cultured from sputum but later typically P. aeruginosa is isolated from lower respiratory tract. Pneumothorax is
seen in >10% of patients.
In infants meconium ileus is seen which leads to abdominal distension, vomiting and failure of passage of stool.
Late onset puberty is common in both males and females.



69. Consider the following statements:
Unconjugated hyperbilirubinaemia occurs predominantly in:
1. Gilbert’s syndrome 2. Alcoholic liver disease
3. Intravascular haemolysis 4. Sickle cell anaemia
Of these statements:
A. 1, 2 and 3 are correct B. 1, 3 and 4 are correct
C. 2, 3 and 4 are correct D. 1 and 4 are correct
Ans. B (Harrison 16th ed., p 241)
Unconjugated hyperbilirubinaemia is seen in Gilbert’s syndrome, intravascular haemolysis and sickle
cell anaemia. In alcoholic liver disease predominant unconjugated hyperbilirubinaemia is not seen. Both direct as
well as indirectly reacting bilirubin are raised. There is intrahepatic cholestasis.
In all types of haemolysis (intravascular as well as extravascular haemolysis) indirectly reacting bilirubin is raised.
Sickle cell anaemia is also a type of haemolytic anaemia.
In Gilbert’s syndrome there is deficiency of hepatic glucoronyl transferase. See Q 74 paper 1 UPSC 2001 for further
details.
Remember the following important points commonly asked in examination:
1. Crigler Najjar Syndrome Type 1 (autosomal recessive) and type 2 (autosomal dominant): Unconjugated
hyperbilirubinaemia.
2. Dubin Johnson syndrome (autosomal recessive): Conjugated hyperbilirubinaemia.
3. Rotor syndrome (autosomal dominant): Conjugated hyperbilirubinaemia.



70. Consider the following statements:
The diagnosis of Wilson’s disease is confirmed by:
1. Serum ceruloplasmin less than 20 mg/dl and the presence of Kayser-Fleischer rings
2. Serum ceruloplasmin more than 20 mg/dl and the presence of Kayser-Fleischer rings
3. Serum ceruloplasmin less than 20 mg/dl and increased hepatic copper above 250 microgram/gram of dry weight
Of these statements:
A. 1 alone is correct B. 2 and 3 are correct
C. 3 alone is correct D. 1 and 3 are correct
Ans. D (Harrison 15th ed., p 2275)
Both the statements 1 and 3 are correct and individually the statements are sufficient for diagnosis of Wilson’s
disease. Majority of these patients have > 10 μg/d copper excretion in urine and have histologic abnormalities in
liver biopsy.

Diagnosis should be suspected in the following cases:
1. Patient below 40 years with unexplained disorder of CNS.
2. Patient below 40 years with features of hepatitis, chronic active hepatitis, unexplained elevation of SGPT and
SGOT.
3. Young patient with unexplained haemolytic anaemia in presence of hepatitis.
4. Young patient with unexplained cirrhosis of liver.



71. Which one of the following is not responsible for concentration of urine in the kidneys?
A. Aldosterone B. Angiotensin II
C. Vasopressin D. Epinephrine
Ans. D (Harrison 16th ed., p 1640-1641)
Although physical factors appear to be major determinants of fluid reabsorption in proximal tubules, hormones (e.g.,
angiotensin II) may also modulate fluid reabsorption directly by enhancing luminal Na+ entry into proximal tubule
cells via an apical Na+/H+ exchanger. Epinephrine is not responsible for concentration of urine.
In certain animals arginine vasopressin (AVP/ADH) enhances NaCl absorption in medullary portion of the thick
ascending limb but whether this occurs in human is uncertain.
Reabsorption of NaCl at collecting tubules is enhanced by aldosterone.



72. Consider the following statements:
Impairment of renal tubular function may cause:
1. Aminoaciduria 2. Decreased phosphate clearance
3. Glycosuria 4. Increased urate clearance
Of these statements:
A. 1, 2 and 3 are correct B. 1, 3 and 4 are correct
C. 2, 3 and 4 are correct D. 1 and 4 are correct
Ans. B (Harrison 16th ed., p 1701)
Renal tubular dysfunction may occur in the form of dysfunction of proximal or distal renal tubules.
Proximal renal tubular dysfunction is known as Fanconi’s syndrome. It is a generalized defect of proximal tubule
and results in proximal renal tubular acidosis, glycosuria with normal blood glucose, hypouricaemia, hypokalaemia,
hypophosphataemia, generalized aminoaciduria and low molecular weight proteinuria.
Fanconi’s syndrome may be inherited as autosomal dominant, recessive or X-linked.
Treatment is with vitamin D and phosphate supplements for healing of bones and alkali supplementation for acidosis.
Salt and water should be taken liberally.


73. In renal tuberculosis, intravenous pyelography may show any of the following except:
A. Calcification B. Cortical atrophy
C. Ulceration of calyces D. Calyceal spasm
Ans. B (Bailey and Love 23rd ed., p 1195)
Intravenous pyelography finding in renal tuberculosis are:
– In the very early stages of the disease the normally clear-cut outline of renal papilla may be rendered indistinct
by ulceration.
– Later, there may be evidence of calyceal stenosis and/or hydronephrosis caused by stricture of renal pelvis or
ureter draining the affected kidney.
– A tuberculous abscess appears as a space-occupying lesion, which causes adjacent calyces to splay out.
– Bladder may appear shrunken with its wall irregularly thickened.
– In late stages there may be dilatation of the contralateral ureter from obstruction where the ureter passes
thickened or oedematous bladder wall.
– Plain abdominal radiograph may show the calcified lesion also known as pseudocalculi.
Remember the following important points about renal tuberculosis:
1. Right kidney affected more than left.
2. Men:Women 2:1.
3. Urinary frequency is earliest symptom and may be only one.

4. Sterile pyuria is seen; in early cases the urine is pale and slightly opalescent.
5. Bacteriological examination of at least three full specimens of early morning urine must be sent for microscopy
and culture.
6. Treatment:
a. Antitubercular chemotherapy is main treatment.
b. Ureteric stenosis: Boari operation or a bowel interposition.
c. Non functioning kidney: Nephroureterectomy.



74. Which of the following are the recognised causes of rapidly progressive glomerulonephritis?
1. Indomethacin 2. Penicillamine
3. Lead 4. Rifampicin
Select the correct answer using the codes given below:
Codes:
A. 2 and 3 B. 2 and 4
C. 1 and 4 D. 1, 2, 3 and 4
Ans. B (Harrison 16th ed., p 1692)
Table: Rapidly progressive glomerulonephritis (RPGN, Crescentic GN).
Type I RPGN:
– Idiopathic.
– Goodpasture syndrome.
Type II RPGN (immune complex):
– Idiopathic.
– Post-infectious.
– Systemic lupus erythematosus.
– Henoch Schonlein purpura (IgA).
Type III RPGN (pauci-immune, ANCA associated):
– Idiopathic.
– Wegener’s glomerulonephritis.
– Microscopic polyarteritis nodosa.
Table: Drug induced glomerulonephritis.
􀁺 RPGN is caused by rifampicin, warfarin, carbimazole, amoxicillin and penicillamine.
􀁺 Minimal change disease (usually with interstitial nephritis) is caused by NSAIDs, recombinant interferon á, rifampicin
and ampicillin.
􀁺 Membranous nephropathy is caused by penicillamine, gold, mercury, trimethadione, captopril and chlormethiazole.
􀁺 Focal segmental glomerulosclerosis is caused by heroin.
􀁺 Pauci-immune necrotizing glomerulonephritis is caused by heroin.
􀁺 Proliferative glomerulonephritis with vasculitis is caused by allopurinol, penicillin, sulfonamides, thiazides and
intravenous amphetamines.
Directions: The next two items are based on the following case history. Study the same carefully and attempt the two items
that follow it.
A 16-year old girl complains of progressive difficulty in climbing stairs and pain in the legs of two months duration. On
examination, she has weakness of neck muscles and proximal muscles of upper and lower limbs. There is mild tenderness of
muscles. Deep tendon reflexes are normal.



75. The most likely diagnosis is:
A. Polymyositis B. Poliomyelitis
C. Muscular dystrophy D. Guillain-Barré syndrome
Ans. A (Harrison 16th ed., p 2540)
This patient is most probably suffering from polymyositis. Poliomyelitis and GB syndrome can be ruled out by
presence of normal deep tendon reflexes. In these two conditions the reflexes will be lost.

Muscular dystrophy is X-linked recessive condition and hence occurs in boys only.
Polymyositis is an inflammatory myopathy. Though it usually affects adults it may be seen in 16 years of age
also. It is called juvenile polymyositis. Proximal muscle weakness with sparing of ocular and facial muscles is the
characteristic feature. Neck flexors are very commonly involved resulting in neck drop. In severe cases respiratory
muscles may be involved.
Treatment is with oral glucocorticoid (prednisolone 1 mg/kg) and if required immunosuppressive drugs
(azathioprine, cyclophosphamide, methotrexate etc) are given.


76. Which one of the following investigations will clinch the diagnosis in this case?
A. Cerebrospinal fluid examination B. Muscle biopsy
C. Nerve conduction studies D. Radiology of cervical spine
Ans. B (Harrison 16th ed., p 2545)
Muscle biopsy is diagnostic of polymyositis. It reveals T cell infiltrates located primarily within the muscle
fascicles and surrounding individual healthy muscle fibres. This leads to phagocytosis and necrosis. In chronic
cases connective tissue is increased and often reacts positively with alkaline phosphatase.
Besides this serum creatinine phosphokinase will be highly raised (but not diagnostic).
EMG will show short duration, low amplitude polyphasic units on voluntary activation and increased spontaneous
activity with fibrillations, positive sharp waves and complex repetitive discharges.
NCV, CSF examination and X-ray cervical spine will be normal.


77. ‘Ataxic nystagmus’ is seen in lesions of:
A. Medial longitudinal fasciculus B. Cerebellum
C. Labyrinth D. Vestibular nuclei
Ans. A
According to Davidson ataxic nystagmus is seen in lesion of medial longitudinal fasciculus (MLF) and ponsmidbrain.
MLF lesion leads to internuclear ophthalmoplegia. MLF links the 6th nerve nucleus in pons with 3rd nerve
nucleus in midbrain.
On examination one eye is slow to adduct (side of lesion) and in the other abducting eye there is coarse horizontal
nystagmus (ataxic nystagmus). It is most commonly seen in multiple sclerosis.
Lesion Type of nystagmus
1. Cerebellum Horizontal nystagmus, maximum on looking towards side of lesion
2. Vestibular apparatus Horizontal or rotatory nystagmus, maximum on looking away from side of lesion
3. Midbrain, superior colliculus Vertical up beating nystagmus that is evoked by looking upwards
4. Lower medulla Down beating nystagmus on downwards or lateral gaze
Pendular nystagmus is seen when ocular fixation is poor. It is usually congenital.


78. Hydrocephalus is a recognised complication of all of the following except:
A. Tuberculous meningitis B. Herpes encephalitis
C. Haemophilus influenzae meningitis D. Arnold-Chiari malformation
Ans. B (Kumar and Clark 4th ed., p 1081)
Hydrocephalus is not a complication of viral encephalitis. Herpes encephalitis is not associated with hydrocephalus.
The characteristic feature of herpes simplex encephalitis is increased signal intensity in frontotemporal, cingulated
or insular regions of brain on T2 weighted spin-echo MRI scan.


79. A raised serum creatine phosphokinase level is unusual in:
A. Acute alcoholic myopathy B. Viral polymyositis
C. Myopathy of Cushing’s syndrome D. Duchenne’s muscular dystrophy
Ans. C (Harrison 16th ed., p 2539)
Raised CK is not seen in steroid induced myopathy. In Cushing’s syndrome myopathy occurs due to excess
corticosteroid. There is proximal muscle weakness with striking muscle wasting.

Biopsy of muscle shows atrophy of muscle fibres. Degeneration and necrosis are absent.
EMG does not show any changes of myopathy.
Remember: Thyrotoxic myopathy is also characterized by normal CK level.



80. Dementia can occur in which of the following cases?
1. Sub-acute spongiform encephalopathy
2. Human immunodeficiency virus infection
3. Cerebral arteriovenous malformation
4. Prion disease
Select the correct answer using the codes given below:
Codes:
A. 1, 2 and 3 B. 2 and 4
C. 1, 2 and 4 D. 3 and 4
Ans. C (Harrison 16th ed., p 2394)
Cerebral arteriovenous malformation leads to CVA in young persons usually. It is not a cause of dementia.
HIV associated dementia or AIDS dementia complex occurs late in HIV infection. Aphasia, apraxia and agnosia
are however not common features in AIDS dementia. Forgetfulness, difficulty of concentration, difficulty in performing
complex tasks are common features. It is an AIDS defining condition.
Creutzfeldt-Jakob (CJD) disease is a prion disease. Prion means proteinaceous infectious particles. The main
feature of CJD is dementia and myoclonus. The patients suffer from memory loss, impaired judgment and decline in
intellectual function. Majority of the patients with clinical features live for 6-12 months only.
SSPE is a progressive and fatal neurological disease caused by measles virus. The onset is slow with intellectual
deterioration, apathy followed by myoclonic jerks, rigidity and dementia. It is probably due to inability of the nervous
system to eradicate the virus.


81. A 22-year old man presents with history of bleeding from gums for the last 6 months. On investigation the Hb was found
to be 8.2 gm%, TLC 4400/mm3, DLC P 64%, L27%, E3%, M6% and platelet count of 20,000/cu mm. Which one of the
following investigations would be most useful in establishing the diagnosis?
A. Bleeding time and clotting time B. Prothrombin time
C. Partial thromboplastin time D. Bone marrow examination
Ans. D
The young patient is suffering from pancytopenia. Haemoglobin is low; TLC and platelet counts are also low. Out
of the given choices bone marrow examination will help to arrive at a diagnosis. It will help to find out the various
causes like aplastic anaemia, aleukaemic leukaemia, megaloblastic anaemia etc.


82. Consider the following steps:
1. Whole blood transfusion 2. Granulocyte transfusion
3. Granulocyte monocyte colony stimulating factor 4. Antibiotics
The management of neutropaenia (WBC less than 500 cells/microlitre) would include:
A. 1, 2 and 3 B. 2 and 3
C. 2, 3 and 4 D. 1 and 4
Ans. C (Harrison 16th ed., p 479)
The management of neutropenia depends upon one important factor i.e., presence or absence of fever. In this
question nothing is mentioned about fever. Fever is a sign of infection and neutropenia patients are at risk of
septicaemia.
Considering these facts whole blood transfusion has no role in neutropenia. It will not serve any purpose. Similarly
Harrison states that granulocyte transfusion has no role because of short half life, mechanical fragility and leucostasis.
But if this choice is removed then answer is not possible because it is mentioned in choice a, b and c.
However, the role of colony stimulating factors like G-CSF and GM-CSF are important because they augment the
production of neutrophils. These are helpful in prolonged neutropenia, neutropenia after chemotherapy etc.
Antibiotics are conventionally used to prevent infection. The choice of antibiotics depends upon several factors like
drug side effect, prevailing infections in community etc.









83. Match List-I (Clinical condition) with List-II (Therapy) and select the correct answer using the codes given below the
Lists:
List-I List-II
a. Acute lymphatic leukaemia 1. Alpha interferon, deoxycoformycin, 2-chlorodeoxyadenosine
b. Acute myelogenous leukaemia 2. Vincristine, prednisone, 1-asparaginase
c. Chronic lymphatic leukaemia 3. Cytarabine, daunorubicin
d. Hairy cell leukaemia 4. Deoxycoformycin, 2-chlorodeoxyadenosine, fludarabine
Codes:
A. a b c d B. a b c d C. a b c d D. a b c d
2 3 1 4 3 2 1 4 3 2 4 1 2 3 4 1
Ans. D
The answer is self explanatory but these regimens have changed slightly in last several years. Hairy cell leukaemia
is responsive to alpha interferon, pentostatin and cladribine. Today cladribine is the preferred drug for management.
Chlorambucil and CHOPP regimen were commonly used for CLL but today fludarabine is the drug of choice
especially in young patients.
The treatment of AML and ALL are more or less unchanged.



84. Thrombocytosis is a recognised feature of:
A. Myelofibrosis B. Systemic lumps erythematosus
C. Azidothymidine therapy D. Myelodysplastic syndrome
Ans. A (Harrison 16th ed., p 629)
Myelofibrosis is a disorder of stem cell characterized by marrow fibrosis, myeloid metaplasia with
extramedullary haematopoiesis and splenomegaly.
Peripheral smear reveals anaemia with teardrop cells, nucleated RBC, myelocytes, promyelocytes and sometimes
myeloblasts. Thrombocytosis is often seen with large and bizarre platelets. Marrow is unaspirable.
There is no specific treatment of myelofibrosis.




85. A 26-year old male presents with weakness, occasional vomiting, hypotension, skin and mucous membrane pigmentation.
The diagnosis can be best established by:
A. Metyrapone test B. Basal plasma cortisol level
C. 24-hour urinary 17-ketosteroid estimation D. ACTH stimulation test
Ans. D (Harrison 16th ed., p 2142)
The patient is suffering from Addison’s disease. It results from progressive destruction of adrenal cortex and
deficiency of glucocorticoid.
Most frequently it is idiopathic (probably autoimmune) but earlier tuberculosis was the commonest cause.
The clinical features are fatigue, weakness, nausea, vomiting, hypotension, mucosal pigmentation, occasionally
hypoglycaemia. Asthenia is the cardinal feature.
Diagnosis is mainly by ACTH stimulation test. Cortisol response is seen 60 minutes after 250 μg of cosyntropin IM
or IV. Cortisol level in normal persons should be more than 18 μg/dl. In Addison’s disease plasma cortisol does not
rise after ACTH.
Treatment is by replacement of glucocorticoid.



86. A young female has polyuria and polydipsia. Daily output of urine is 7 litres. Intake of fluids is 8 litres. On examination
urine is clear and alkaline with a specific gravity of 1001. There is no glycosuria or proteinuria. Specific gravity of the
urine after restriction of water from 7 PM to 7 AM is 1012. The most likely diagnosis is:
A. Diabetes insipidus B. Nephrogenic diabetes insipidus
C. Chronic interstitial nephritis D. Compulsive polydipsia
Ans. D (Harrison 16th ed., p 2099-2102)
The patient is suffering from compulsive polydipsia. The main test to distinguish between diabetes insipidus and
polydipsia is water deprivation test.
After 12 hours of water deprivation the specific gravity has increased to 1012 and this means severe pituitary or
nephrogenic diabetes insipidus is ruled out.









87. In respect of Turner’s syndrome, all of the following are true except:
A. Atrial septal defect is the common cardiac abnormality
B. Congenital renal abnormalities are often associated
C. Birth weight is significantly below that of a normal infant
D. Osteoporosis is very often seen
Ans. A (Nelson 16th ed., p 1753; Harrison 15th ed., p 1333)
Coarctation of aorta, bicuspid aortic valve and aortic dilatation are the common cardiac malformations in Turner’s
syndrome.
Many patients with Turner’s syndrome are recognizable at birth because of characteristic oedema of the dorsum of
hands and feet and loose skin folds at the nape of the neck.
– Low birth weight and decreased length are common.
– Clinical manifestation in childhood include webbing of the neck, a low posterior hairline, small mandible, prominent
ears, epicanthal folds, high arched palate, a broad chest presenting the illusion of widely spaced nipples,
cubitus valgus and hyperconvex fingernails.
– Diagnosis is first suspected at puberty when sexual maturation fails to occur.
– One fourth to one third of patients have renal malformations on ultrasonographic examination. The more serious
defects include pelvic kidney, horseshoe kidney, double collecting system, complete absence of one kidney and
ureteropelvic junction obstruction.



88. In terms of chemical structure, which one of the following is a tetrameric glycoprotein?
A. Insulin receptors B. Leptins
C. Neuropeptide D. Thyrotrophin
Ans. A (Lippincott 2nd ed., p 273)
Insulin receptor is synthesized as single polypeptide that is glycosylated and cleaved into alpha and beta subunits,
which are then assembled into a tetramer linked by disulfide bonds.
Alpha unit contains insulin-binding site.
Beta subunit is a tyrosine kinase, which is activated by insulin.
Insulin dependent glucose transport is seen in skeletal muscles, adipocytes etc.
Insulin independent glucose transport is seen in hepatocytes, erythrocytes, cells of the nervous system, intestinal
mucosa, renal tubule and cornea.



89. Consider the following statements:
Hyponatraemia is a feature of:
1. Thiazide diuretics therapy 2. Glucocorticoid deficiency
3. Hyperaldosteronism 4. Infective diarrhoea
Of these statements:
A. 1, 2 and 3 are correct B. 2 and 4 are correct
C. 1, 3 and 4 are correct D. 1, 2 and 4 are correct
Ans. D (Harrison 16th ed., p 255)
Thiazide diuretics cause hyponatraemia by causing natriuresis.
Diarrhoea causes gastrointestinal loss of Na+ ions and thus causes hyponatraemia.
Glucocorticoid deficiency causes hypoosmolal hyponatraemia by primary water gain (secondary Na+ loss).
Hypoaldosteronism causes hyponatraemia.




90. Arterial blood gas data in a young girl on admission shows the following changes:
PO2 104 mmHg,
O2 saturation 99%,
PCO2 24 mmHg,
HCO3
12 and
pH = 7.25.

The most likely diagnosis is:
A. Acute severe bronchial asthma B. Diabetic ketoacidosis
C. Hysterical hyperventilation D. Ethanol ingestion
Ans. B
The most likely diagnosis is diabetic ketoacidosis. The picture is of metabolic acidosis (pH 7.25, HCO3
12).
In acute severe bronchial asthma there should be respiratory acidosis. PO2 should be low. Normal HCO3
level is
22-25 mEq/l.
Ethanol ingestion will not have similar change.




91. Consider the following statements:
Increased oxalate crystal in urine is seen in:
1. Pyridoxine deficiency 2. Chronic ileal disease
3. Primary hyperoxaluria
Of these statements:
A. 2 and 3 are correct B. 1 and 3 are correct
C. 1 and 2 are correct D. 1, 2 and 3 are correct
Ans. D (API Textbook of Medicine p 253; Schwartz Principles of Surgery 6th ed., p 745)
Intestinal disease such as chronic diarrhoeal disease, ileal disease or intestinal resection may predispose the
patient towards enteric hyperoxaluria or hypocitraturia resulting in calcium oxalate stones.
– In cases of positive family history for renal stone the following causes should be ruled out e.g., absorptive
hypercalciuria, cystinuria, renal tubular acidosis and primary hyperoxaluria due to increased calcium, cysteine
or oxalate in urine.
According to API hyperoxaluria also occurs due to pyridoxine deficiency and chronic small intestinal disease.
There is no specific treatment for hyperoxaluria. Forced alkaline diuresis, high phosphate feeding, oral magnesium
oxide and large doses of pyridoxine have been advocated.




92. Tetany may be present in all the following conditions except:
A. Acute pancreatitis B. Hysterical hyperventilation
C. Hyperkalaemia D. Hypomagnesemia
Ans. C
Tetany is not seen in hyperkalaemia. Tetany is seen in hypocalcaemia, alkalosis and hypomagnesaemia.
Acute pancreatitis causes hypocalcaemia and hence tetany may be seen.
In hysterical hyperventilation due to decrease in ionic calcium patient develops tetany.



93. In patient of acquired immunodeficiency syndrome, the commonest cause of space occupying lesion in brain is:
A. Non-Hodgkin’s lymphoma B. Cytomegalovirus infection
C. Cryptococcosis D. Toxoplasmosis
Ans. D (Harrison 16th ed., p 1117-1118)
According to Harrison a study was conducted to find out the cause of seizure in HIV infected persons. Cerebral
mass lesion was the commonest cause and out of this majority was due to toxoplasmosis.
Toxoplasmosis is a late complication of AIDS. It occurs when CD4+ T cell count is < 200/μL. Commonest presentation
is in the form of fever, headache and focal neurological deficit. MRI shows multiple lesions at multiple locations.
Standard treatment is sulfasalazine and pyrimethamine with leucovorin for 4 to 6 weeks.
Prophylaxis is with one double strength tablet of trimethoprim and sulfamethoxazole.
Remember: Immunoblastic lymphoma is the commonest lymphoma (60%) in AIDS. Burkitt’s lymphoma is seen in
20% cases and primary CNS lymphoma is seen in 20% cases. About 90% of lymphomas are of B cell phenotype
and half contain EBV DNA.



94. Rickettsia burnetii is the causative organism of:
A. Trench fever B. Q fever
C. Epidemic typhus D. Scrub typhus

Ans. B (Harrison 16th ed., p 1007)
Rickettsia burnetii causes Q fever. Now the causative agent is called C. burnetii. It is a small gram-negative
microorganism.
Acute Q fever presents with flu like features with fever, pneumonia, hepatitis, pericarditis, myocarditis,
meningoencephalitis etc.
Chronic Q fever presents with endocarditis.
Treatment is with doxycycline 100 mg BD for 2 weeks. In chronic cases rifampicin 300 mg OD is also given. The
drugs are given for 3 years.
Table: Rickettsial diseases of human.
Group Species Disease Vector
Spotted fever group R. rickettsii Rocky Mountain spotted fever Tick
R. siberica Siberian tick typhus Tick
R. conori Boutonneuse fever Tick
R. conori Indian tick typhus Tick
R. conori South African tick typhus Tick
R. conori Kenyan tick typhus Tick
R. akari Rickettsial pox Gamasid mite
Typhus group R. prowazekii Epidemic typhus Louse
R. prowazekii Brill-Zinsser disease —
R. mooseri Endemic typhus Rat flea
Scrub typhus R. tsutsugamushi Scrub typhus Trombiculid mite
Q fever Cox. burnetii Q fever Human-nil
Extra human-tick
Trench fever Ro. quintana Trench fever Louse



95. Consider the following statements:
Both Corynebacterium diphtheriae and Clostridium tetani are:
1. Spore forming bacilli 2. Gram positive bacilli
3. Aerobic bacilli 4. Exotoxin producing bacilli
Of these statements:
A. 1 and 3 are correct B. 2 and 4 are correct
C. 1, 2 and 3 are correct D. 2, 3 and 4 are correct
Ans. B (Harrison 16th ed., p 835, 845)
Both Corynebacterium diphtheriae and Clostridium tetani are gram-positive bacilli and produce exotoxin.
Clostridium tetani causes tetanus. It is anaerobic and forms spores, which are resistant to various disinfectants. It
produces tetanospasmin, which blocks the neurotransmitters glycine and GABA at presynaptic terminals.
C. diphtheriae is aerobic, nonmotile, non-sporulating, irregularly staining gram-positive bacilli. It is the causative
agent of diphtheria.
The diphtheria toxin is a protein and is extremely potent. It has two fragments A and B. All the enzymatic activity of
toxin is present in fragment A. Fragment B is responsible for binding of the toxin to the cells. Toxin production is
influenced by the concentration of iron in the medium. Optimum level of iron for toxin production in 0.1 mg/l.




96. A 34-year old person has rapidly developing cough, dyspnoea, expectoration and blood-tinged sputum. He is febrile,
cyanosed and toxic. Chest examination reveals crepitations and rhonchi. The most likely diagnosis is:
A. Legionella pneumonia B. Pneumonic plague
C. Septicaemic plague D. Pulmonary tuberculosis
Ans. B (Harrison 16th ed., p 924)
The clinical presentation described is typical of pneumonia. There is however very little clue about the aetiological
agent.

Pulmonary TB is ruled out because the history is short and the patient is toxic.
Septicaemic plague often presents with gastrointestinal symptoms with features of DIC and shock. Hence it is
also ruled out.
Legionella pneumonia is a type of community acquired pneumonia and this patient well may be a case of it but
usually legionella pneumonia is a type of atypical pneumonia. Toxicity and cyanosis are not common. Besides that
it is considered to be associated with air coolers and humidifiers. However latest observation reveals that it is
transmitted through drinking water as well. But such clues are absent.
Considering the above facts it is probably a case of pneumonic plague. It is caused by Y. pestis. The onset is
sudden with chills, fever, myalgia, weakness etc. Cough with expectoration, dyspnoea, blood tinged sputum and
cardiopulmonary insufficiency rapidly occurs along with circulatory collapse.
Streptomycin, gentamicin, tetracycline and chloramphenicol are used for treatment.




97. Which one of the following is the most contagious lesion in syphilis?
A. Primary sore B. Gumma
C. Condylomata D. Papular skin rash
Ans. C (Short Textbook of Dermatology and Venereology 1st ed., Praveen Jain p 95)
Condylomata lata is a warty plaque like lesion found in perianal area and other moist body sites. It is seen in
secondary syphilis and is the most contagious lesion. It heals without treatment.
In secondary syphilis most common organ involved is skin and most common lesion is papulosquamous rash.
Remember the following important points commonly asked in examination:
􀁺 Condyloma acuminata (genital wart) is caused by HPV mainly 6, 11 (most common). Incubation period is 2-
4 months. It presents as soft fleshy growth. Cryotherapy is treatment of choice. Also useful are podophyllin
20%, podophyllotoxin 0.5% (for self application). Podophyllin is not used in pregnancy.



98. Which one of the following is not a complication of visceral leishmaniasis?
A. Pulmonary tuberculosis B. Renal cortical necrosis
C. Cancrum oris D. Epistaxis
Ans. B (WHO Expert Committee Report, 1991; emedicine.com)
All the four mentioned choices can be complication of visceral leishmaniasis. However the best answer is renal
cortical necrosis. Pulmonary TB can occur due to depressed immunity in visceral leishmaniasis.
Renal cortical necrosis may occur due to thrombosis of blood vessels supplying the renal cortex. However this is
not very common. Tubular necrosis is more commonly seen as compared to cortical necrosis. Cancrum oris is also
a well known complication. Haemorrhagic complications occur due to thrombocytopenia. Dear reader if you think
otherwise please contact the publisher with reference.



99. All of the following are features of cerebral malaria except:
A. Hyperglycaemia B. Thrombocytopaenia
C. Acute respiratory distress syndrome D. Heavy parasitaemia
Ans. A (Harrison 16th ed., p 1222)
Hypoglycaemia occurs in falciparum malaria. Hyperglycaemia is not seen. Hypoglycaemia results from failure
of hepatic gluconeogenesis and increased consumption by both the parasite and the host. It is specially a problem
in children and pregnant women.
Remember the following important points commonly asked in examination:
􀁺 Quinine and quinidine used to treat cerebral malaria cause hypoglycaemia by stimulating release of insulin.
􀁺 The other complications of severe malaria are renal failure, DIC, hypotension, acidosis, ARDS, convulsions,
haemoglobinuria, jaundice etc.



100. Match List-I (Adverse reaction) with List-II (Drug) and select the correct answer using the codes given below the lists:
List-I List-II
a. Drug induced lupus 1. Procainamide
b. Toxic epidermal necrolysis 2. Clofazimine
c. Scleroderma-like syndrome 3. Pentazocine
d. Red discoloration 4. Thiacetazone

Codes:
A. a b c d B. a b c d C. a b c d D. a b c d
1 4 2 3 4 1 2 3 4 1 3 2 1 4 3 2
Ans. D
This question hardly needs any explanation, as the question is self-explanatory.



101. Ethosuximide is an effective drug for the treatment of:
A. Tonic-clonic generalized seizures B. Focal seizures
C. Absence seizures D. Myoclonic seizures
Ans. C (Harrison 16th ed., p 2367)
Ethosuximide (20-40 mg/kg) is very effective in management of absence seizures. It is not effective against
other types of seizures.



102. Consider the following conditions:
1. Plague 2. Hansen’s infection
3. Legionellosis 4. Resistant staphylococcal infection
Indications for rifampicin therapy would include:
A. 1 and 3 B. 2 and 4
C. 1, 2 and 3 D. 2, 3 and 4
Ans. D (Harrison 16th ed., p 800, 961, 965)
Rifampicin is used in Hansen’s infection. It is the most active agent in leprosy. It is bactericidal against M. leprae. It
is given 600 mg once every month. For legionella infection rifampicin is given in the dose of 300-600 mg BD for 2
weeks.
Rifampicin is used in staphylococcal prosthetic valve endocarditis. It is a reserved drug for refractory, relapsing or
inoperable S. aureus infection.



103. Which one of the following fungi causes mycetoma of the foot?
A. Histoplasma capsulatum B. Cryptococcus neoformans
C. Epidermophyton floccosum D. None of the above
Ans. D (Harrison 16th ed., p 1192)
Mycetoma is caused by actinomycetes of the genera Nocardia, Nocordiopsis, Streptomyces and Actinomadura.
It is a chronic suppuration of subcutaneous tissue characterized by presence of grains that are clumped colonies of
causative agent. Sinus tract develops and pus is discharged intermittently.
Treatment is with streptomycin and either dapsone or trimethoprim and sulfamethoxazole.



104. Consider the following conditions:
1. Diabetes mellitus 2. Cushing’s syndrome
3. Human immunodeficiency virus infection 4. Post-organ transplantation
Oral thrush is commonly seen in:
A. 1, 2 and 3 B. 2 and 4
C. 3 and 4 D. 1, 3 and 4
Ans. D
Oral thrush is usually due to candida infection. It is commonly seen in immunosuppressed conditions.
Amongst the given choices diabetes, HIV infection and post organ transplantation (on immunosuppressive drugs)
are complicated by oral thrush.
Oral thrush also occurs in patients receiving corticosteroid, chemotherapy etc.


105. The likely metabolic complications of total parenteral nutrition within the first two weeks of therapy would include all of
the following except :
A. Cardiopulmonary failure B. Hyperosmolar non-ketotic hyperglycaemia
C. Zinc deficiency D. Hypophosphataemia

Ans. C (Bailey and Love 24th ed., p 82)
In first 48 hours of TPN, the metabolic complications noticed are fluid overload, hyperglycaemia, hypokalaemia and
hypophosphataemia. Till 2 weeks the complications seen are cardiopulmonary failure, hyperosmolar nonketotic
hyperglycaemic coma, acid base imbalances and electrolyte imbalance (hyponatraemia, hypomagnesaemia and
hyperammonaemia). Late metabolic complications include cholestatic liver disease (with bile sludging and gall
stone formation) and bone disease (osteopenia and calciuria).


106. Consider the following features:
1. Binge eating 2. Hypokalaemia
3. Hypothermia 4. Amennorrhoea in half of the patients
The recognized features of bulimia would include:
A. 1, 2 and 4 B. 1 and 3
C. 1, 3 and 4 D. 2 and 4
Ans. A (Harrison 16th ed., p 431)
Binge eating is the main feature of bulimia nervosa. It occurs in a discreet period of time and during it there is
a lack of control over eating. Between the binges the patient tries for restriction of calories. They develop the
capacity to induce vomiting at will. Repeated vomiting may cause alkalosis.
The incidence of amenorrhoea and oligomenorrhoea is high.
Hypokalaemia, hypochloraemia and hyponatraemia may be seen.
Hypothermia is absent.
Cognitive behaviour therapy and antidepressants are used for management of bulimia nervosa.


107. A mentally retarded child aged 12 years has multiple, painful, discharging shiny white lesions around the anus. Which of
the following is the most probable diagnosis?
A. Lupus vulgaris B. Carcinoma
C. Syphilitic condyloma D. Haemorrhoids
Ans. C
The child seems to be suffering from syphilitic condyloma. He seems to be abused sexually because of mental
retardation and the clinical picture described does not match with lupus vulgaris, carcinoma and haemorrhoids.



108. All of the following drugs can cause photodermatitis except:
A. Griseofulvin B. Chloroquine
C. Captopril D. Oral contraceptives
Ans. B (Harrison 15th ed., p 108)
Chloroquine does not cause photodermatitis. The various other drugs that cause photodermatitis are
chlorediazepoxide, furosemide, NSAID, sulfonamides, tetracycline, thiazide, phenothiazines etc.


109. Consider the following:
1. Cutaneous lupus
2. Erythropoietic porphyria
3. Lichen planus
4. Retrovirus infection
Causes of scarring alopecia would include:
A. 3 and 4 B. 1, 2 and 3
C. 2, 3 and 4 D. 1 and 4
Ans. B (Harrison 16th ed., p 298)
In scarring alopecia there is inflammation, fibrosis and loss of hair follicles. The various other causes are folliculitis
declavans, morphea, trauma, sarcoidosis, lupus erythematosus, cutaneous metastasis etc.
In non-scarring alopecia the hair shafts are lost but hair follicles are intact. The various causes are telogen
effluvium, alopecia areata, tinea capitis, androgenic alopecia, trauma, drugs, lupus erythematosus, hypothyroidism,
HIV infection, secondary syphilis etc.







110. A 16-year old female has recurrent episodes of both large and small joint pain with photosensitive skin rashes all over the
body. Recently she has developed abnormal behaviour. Her cerebrospinal fluid was positive for antineuronal antibodies.
The most likely diagnosis is:
A. Rheumatic fever with chorea B. Systemic lupus erythematosus with organic brain syndrome
C. Systemic necrotising vasculitis D. Chronic angioedema
Ans. B (Harrison 16th ed., p 1963; Neuro Px Vol 2(2):June 2002)
The patient is suffering from SLE. Points in favour of diagnosis are:
1. Young female.
2. Joint pain.
3. Photosensitive skin rash.
CNS disease correlates with elevated levels of antineuronal antibodies. 30-90% patients with active CNS disease have
elevated levels of antineuronal antibodies. They are thought to originate either in blood and cross blood brain barrier
or to be synthesized in situ from B cells within the CNS.


111. Behcet’s disease would include which of the following?
1. Recent aphthous ulcers of the mouth 2. Recurrent genital ulceration
3. Arthritis of wrist and metacarpophalangeal joints 4. Uveitis and optic neuritis
Select the correct answer using the codes given below:
Codes:
A. 1 and 2 B. 2, 3 and 4
C. 1, 3 and 4 D. 1, 2 and 4
Ans. D (Harrison 16th ed., p 2014)
Behcet’s syndrome is a multisystem disorder. The diagnostic criteria are:
1. Recurrent genital ulcers.
2. Skin lesions.
3. Eye lesions.
4. Pathergy test.
The disease mainly affects young adults. The recurrent oral aphthous ulcers are painful and persists for about 1-2
weeks without scars. The genital ulcers are similar to oral ulcers.
Skin lesions are folliculitis, erythema nodosum, acne like exanthem etc.
Pathergy test: Nonspecific skin reactivity to intradermal saline injection.
Eye lesions are iritis, posterior uveitis, retinal vessel occlusion etc. Hypopion uveitis is the hallmark of Behcet’s
syndrome but is rare.


112. Consider the following features:
1. Dementia 2. Pathological jealousy
3. Good insight 4. Suicidal tendency
Psychiatric manifestations of alcohol abuse would include:
A. 1, 2 and 3 B. 3 and 4
C. 1, 2 and 4 D. 2 and 4
Ans. C (Kumar and Clark 4th ed., p 1114, 1136)
Psychological problems caused by alcohol abuse are:
– Depression.
– Attempted and complete suicidal tendency.
– Pathological jealousy.
– Personality disturbances.
– Wernicke’s encephalopathy.
– Korsakoff’s psychosis.
– Delirium tremens.
– Anxiety and phobias.
– Memory disturbances.
– Dementia.







113. Match List-I with List-II and select the correct answer using the codes given below the Lists:
List-I List-II
a. Panic disorder 1. Hypertonic muscles, somnolence and ataxia
b. Lithium effect 2. Depressed mood, low self-esteem
c. Chronic fatigue syndrome 3. Dyspnoea, palpitation and trembling
d. Dysthymia 4. Photophobia, rapid pulse and night sweat
Codes:
A. a b c d B. a b c d C. a b c d D. a b c d
3 1 4 2 4 3 2 1 3 2 4 1 4 1 3 2
Ans. A (Harrison 15th ed., p 2541, 2543, 2549)
The best possible answer is A but the picture of chronic fatigue syndrome is not clear in list II.
Dysthymic disorder consists of pattern of chronic (at least 2 years) ongoing, mild depressive symptoms that are
less severe and less disabling than those found in major depression and the two conditions are sometimes difficult
to separate.
When the two occur together then it is called double depression.
Diagnostic criteria for panic disorder:
A period of intense fear or discomfort in which 4 or more of the following symptoms developed abruptly and reach a
peak within 10 min.
1. Palpitation, pounding heart or accelerated heart rate.
2. Sweating.
3. Trembling/shaking.
4. Sensation of shortness or smothering.
5. Feeling of choking.
6. Chest pain or discomfort.
7. Nausea or abdominal distress.
8. Feeling dizzy, unsteady, lightheaded or faint.
9. Derealization (feeling of unreality) or depersonalization being detached from oneself.
10. Fear of losing control or going crazy.
11. Fear of dying.
12. Paresthesias (numbness or tingling sensations).
13. Chills or hot flushes.



114. Which of the following are the eye findings in a case of trisomy 21 (Down’s syndrome)?
1. Palpebral fissure slanting downward medially with prominent epicanthic folds
2. Brushfield’s spots on iris
3. Dislocation of lens
4. Aphakia
Select the correct answer using the codes given below:
Codes:
A. 1 and 2 B. 2 and 3
C. 3 and 4 D. 1 and 4
Ans. A (Nelson 16th ed., p 1919)
Eye findings in case of trisomy 21 (Down syndrome):
– Mongoloid slant of palpebral fissure – Keratoconus and corneal hydrops
– Epicanthus – Cataract
– Dacryostenosis – High refractive errors
– Blepharitis – Strabismus
– Brushfield spots of iris – Nystagmus
– Peripheral thinning of iris stroma – Increased vessels at disc
Dislocation of lens and aphakia are not seen in Down syndrome.







115. Recombinant DNA technique is used in the preparation of which of the following?
1. Porcine insulin 2. Human insulin
3. Antirabies vaccine 4. Hepatitis B vaccine
Select the correct answer using the codes given below:
Codes:
A. 1 and 3 B. 2 and 4
C. 1, 2 and 3 D. 2, 3 and 4
Ans. B (Harper p 498; Essentials of Medical Biochemistry 1st ed., RC Gupta p 326-233; hHp://www.utoronto.ca)
Porcine insulin is not prepared by recombinant DNA technology. Antirabies vaccine is prepared by cell culture
technology.
Recombinant DNA technology has 2 distinct merits in production of material:
1. It can supply large amount of material that could not be obtained by conventional purification methods.
2. It can provide human material e.g., insulin, growth hormone.
Primary aim is to supply products, generally proteins for treatment (insulin) and diagnosis (AIDS test) of human and
other animal diseases and for disease prevention (hepatitis B vaccine).
After extensive review of literature we have come across a single mention of recombinant antirabies vaccine under
development at Egypt which requires 6 shots.
Applications of recombinant DNA technology:
1. Mapping of genomes.
2. Production of proteins: Proteins of diagnostic, therapeutic, nutritional and industrial importance can be produced
in large quantities by recombinant DNA technology e.g., human insulin, human growth hormone, interferon,
tissue plasminogen activator, factor VIII, erythropoietin, bovine growth hormone and subtilisin (proteolytic enzyme
used in detergents). Hepatitis B vaccine is also prepared in yeast as a subunit vaccine.
3. Diagnosis of genetic diseases.
4. Medicolegal application by DNA finger prints.
5. Gene therapy.



116. Match List-I (Poisons) with List-II (Antidotes) and select the correct answer using the codes given below the lists:
List I List II
a. Iron 1. Acetylcysteine
b. Benzodiazepines 2. PAM (Pyridine-2 aldoxy-methiodate)
c. Acetaminophen 3. Desferrioxamine
d. Organophosphorus 4. Flumazenil
Codes:
A. a b c d B. a b c d C. a b c d D. a b c d
4 3 1 2 3 4 2 1 4 3 2 1 3 4 1 2
Ans. D
􀁺 Desferrioxamine is a specific iron-chelating agent. It is given IV 10-15 mg/kg/h. It may cause hypotension
and pulmonary oedema.
􀁺 Flumazenil is benzodiazepine receptor antagonist. 0.1 mg is given IV at 1-minute interval till desired effect
is achieved. It has a short half-life and hence one should be careful about relapse of toxicity.
􀁺 N-acetylcysteine is used in acetaminophen toxicity. It is given in a loading dose of 140 mg/kg followed by 40
mg/kg every 4 hour for 17 doses.
􀁺 PAM is a cholinesterase reactivator and is useful to relieve nicotinic features of organophosphorous
poisoning. Its use in carbamate poisoning is doubtful. 1-2 g is given slow IV and can be repeated if response is
incomplete.



117. Consider the following features:
1. Convulsion followed by coma 2. Blood leaving the lung is supersaturated with oxygen
3. Absorption atelectasis in the lung 4. Symptoms occur immediately
The features of oxygen poisoning would include:
A. 1 and 3 B. 2 and 4
C. 1, 2 and 3 D. 1, 2, 3 and 4

Ans. A (Short Textbook of Anaesthesia 2nd ed., Ajay Yadav p 69; Miller Anesthesia 5th ed., p 612-613, 2274; Kumar
and Clark 4th ed., p 889)
Prolonged high concentrations of oxygen can cause pulmonary toxicity. 100% oxygen is usually considered safe for
up to 8-12 hours in normal adults (in infants and neonates 100% oxygen for more than 2-3 hours can cause
pulmonary toxicity). Oxygen toxicity is because of toxic radicals of oxygen like superoxide and hydroxyl ions,
singlet oxygen, hydrogen peroxide.
Convulsions followed by coma is the most acute toxic enzyme effect of O2 (Paulbert effect). It is dose dependent
and the dose depends on the partial pressure of O2 inspired and length of exposure.
Absorption atelectasis, abnormalities of ciliary transport and tracheobronchitis are features of O2 toxicity. In neonates
pulmonary toxicity manifests as bronchopulmonary dysplasia. In neonates there is risk of retrolental fibroplasia also.
Virtually complete saturation of hemoglobin with O2 occurs at a PO2 of 100 mmHg and further increase in PO2 does
not alter the quantity of O2 bound to Hb. However there is increase in quantity of O2 dissolved in plasma.



118. Which one of the following is not a laboratory feature of familial paroxysmal polyserositis (familial Mediterranean
fever)?
A. Polymorphonuclear leukocytosis B. Elevated erythrocyte sedimentation rate
C. Elevated plasma lipids D. Non-specific ECG changes
Ans. C (Harrison 16th ed., p 1794)
Familial Mediterranean fever (FMF) is an autosomal recessive disorder characterized by recurrent episodes
of fever, peritonitis and/or pleuritis. It predominantly occurs in non-Ashkenazi Jews.
It is due to mutation in MEFV gene located on chromosome 16p. It encodes pyrin (amino acid) expressed in cells
of myeloid lineage.
FMF occurs between ages of 5-15 years and the usual attack lasts for 1-2 days but may be prolonged. Patient may
have fever, abdominal pain, chest pain, joint pains, skin lesions etc.
Plasma lipids are normal and rest of the mentioned laboratory features are seen.
Treatment is supportive. Prophylaxis is with colchicine.



119. Gingival biopsy is useful in the diagnosis of:
A. Sarcoidosis B. Amyloidosis
C. Histoplasmosis D. Scurvy
Ans. B (Harrison 16th ed., p 2027)
Amyloidosis is diagnosed by biopsy and the common sites for taking biopsy samples are gingiva, rectum,
subcutaneous abdominal fat, skin etc.
Remember the following important points commonly asked in examination:
􀁺 Congo red stain: Stains red. It has unique green birefringence when seen through polarizing microscope.
􀁺 HE stain: Amyloid is stained pink.
􀁺 Crystal violet stain: Shows metachromasia.
􀁺 Amyloid syndrome due to â2 microglobulin deposition as amyloid fibrils is seen in patients on long term
haemodialysis.
􀁺 Scintigraphy using 123I-labelled serum amyloid P component is useful for the assessment of AL, ATTR and AA
amyloidosis.



120. Plasmapheresis is recommended in all of the following except:
A. Hyperviscosity syndrome B. Macroglobulinaemia
C. Immune complex glomerulonephritis D. Chronic active hepatitis
Ans. D (Harrison 16th ed., p 1682-1684)
Plasmapheresis (plasma exchange) is done to remove some harmful substance (e.g., harmful antibodies) from
blood. Chronic active hepatitis is due to hepatitis B or C viruses, which are present in liver cells mainly and destroy
them progressively. Hence by doing plasmapheresis no benefit will be there. It is not recommended in management
of chronic active hepatitis.
Besides the diseases mentioned in the questions it is also used in myasthenia gravis and Goodpasture’s
syndrome.

No comments:

Post a Comment